krok 2 - 2012 question paper (stomatology)

28
Krok 2 Stomatology 2012 1 1. A 49-year-old patient complains about pain in the oral cavity induced by eating. He suffers from CHD. Objecti- ve examination revealed dyspnea, limb edema. Oral cavity isn’t sanitated. On the mucous membrane on the right, there is an ulcer with irregular edges covered with greyish-white necrotic deposit with low- grade inammation around it. There is also halitosis. What is the most probable diagnosis? A. Trophic ulcer B. Traumatic ulcer C. Tuberculous ulcer D. Cancerous ulcer E. Ulcero-necrotic stomatitis 2. A 48-year-old patient complains about itching gums. Objectively: gums are dense, of normal colour; cold water causes short-term toothache. X-ray pi- cture shows osteosclerosis of interalveolar septa (ne granular bone texture), hei- ght of interalveolar septa and integrity of compact substance on their tops is unchanged. Which diagnosis is the most likely? A. Initial parodontosis B. Atrophic gingivitis C. Initial periodontitis D. I degree periodontitis E. I degree parodontosis 3. A 34-year-old patient had got in a car accident. The patient stayed consci- ous. He complains of headache, dizzi- ness, general weakness, nausea. 12 hours after the injury the patient got "raccoon eyes"(periorbital haematomas) within the circular muscles of eye. What fracture does the victim have? A. Fracture of skull base B. Le Fort II fracture of maxilla C. Fracture of nasal bones D. Bilateral fracture of zygomatic bones E. Fracture of frontal bones 4. A victim got a perforating wound of the left cheek contaminated with radioactive materials on the battleeld. The wound was dressed with aseptic bandage from the individual rst-aid pack and processed with anaesthetic and antibacterial medicaments. The dressing should be changed at the following stati- on of medical evacuation: A. At each station of evacuation B. At the battalion aid station C. At the regimental aid station D. At the station of the separate medical battalion E. Only when the dressing is unsatisfactory 5. A 47-year-old female patient complai- ns of inability to eat hot and cold food, as well as of intense pain caused by sour, sweet and salty food. Objectively: there is a slight loss of enamel on the molars and incisors. Probing and cold test cause acute pain. What is the most likely diagnosis? A. Hyperesthesia of dental hard tissues B. Enamel erosion C. Enamel necrosis D. Enamel hypoplasia E. Pathological abrasion of dental hard tissues 6. After removal of dental plaque an 18- year-old patient underwent preventive examination. It revealed painless chalky spots in the precervical region on the vestibular surface of the 22 and 41 teeth. Result of enamel resistance test is 7. What morphological changes are typical for this disease? A. Subsurface enamel demineralization B. Changes in the mantle dentine C. Damage of dentinoenamel junction D. Supercial enamel demineralization E. Degeneratic changes of odontoblasts 7. A 49-year-old patient complains of constant mandibular pain irradiating to the ear, soft tissue edema, body temperature rise up to 39, 0 o C . Objecti- vely: there is an edema in the submandi- bular region, the skin has not changed in colour. Alveolar mucosa around the 45, 46 teeth is hyperemic and edematic on the oral and vestibular side. Crowns of the 45, 46 teeth are completely destroyed, the teeth are mobile. Vincent’s symptom is present. What is the most likely diagnosis? A. Acute mandibular osteomyelitis B. Abscess of submandibular region C. Exacerbation of chronic mandibular osteomyelitis D. Acute serous periostitis of mandible E. Acute purulent periostitis of mandible 8. An 8-year-old boy complains of havi- ng toothache during eating. Objectively: approximal surface of the 55 tooth has a deep carious cavity communicating wi- th the tooth cavity. Probing causes acute pain, there is bleeding, percussion is pai-

Upload: eneutron

Post on 14-Apr-2017

462 views

Category:

Health & Medicine


4 download

TRANSCRIPT

Page 1: Krok 2 - 2012 Question Paper (Stomatology)

Krok 2 Stomatology 2012 1

1. A 49-year-old patient complainsabout pain in the oral cavity induced byeating. He suffers from CHD. Objecti-ve examination revealed dyspnea, limbedema. Oral cavity isn’t sanitated. On themucous membrane on the right, there isan ulcer with irregular edges covered withgreyish-white necrotic deposit with low-grade inflammation around it. There isalso halitosis. What is the most probablediagnosis?

A. Trophic ulcerB. Traumatic ulcerC. Tuberculous ulcerD. Cancerous ulcerE. Ulcero-necrotic stomatitis

2. A 48-year-old patient complains aboutitching gums. Objectively: gums aredense, of normal colour; cold watercauses short-term toothache. X-ray pi-cture shows osteosclerosis of interalveolarsepta (fine granular bone texture), hei-ght of interalveolar septa and integrityof compact substance on their tops isunchanged. Which diagnosis is the mostlikely?

A. Initial parodontosisB. Atrophic gingivitisC. Initial periodontitisD. I degree periodontitisE. I degree parodontosis

3. A 34-year-old patient had got in acar accident. The patient stayed consci-ous. He complains of headache, dizzi-ness, general weakness, nausea. 12 hoursafter the injury the patient got "raccooneyes"(periorbital haematomas) within thecircular muscles of eye. What fracturedoes the victim have?

A. Fracture of skull baseB. Le Fort II fracture of maxillaC. Fracture of nasal bonesD. Bilateral fracture of zygomatic bonesE. Fracture of frontal bones

4. A victim got a perforating woundof the left cheek contaminated withradioactive materials on the battlefield.The wound was dressed with asepticbandage from the individual first-aidpack and processed with anaesthetic andantibacterial medicaments. The dressingshould be changed at the following stati-on of medical evacuation:

A. At each station of evacuationB. At the battalion aid stationC. At the regimental aid stationD. At the station of the separate medicalbattalionE. Only when the dressing is unsatisfactory

5. A 47-year-old female patient complai-ns of inability to eat hot and cold food,as well as of intense pain caused by sour,sweet and salty food. Objectively: there isa slight loss of enamel on the molars andincisors. Probing and cold test cause acutepain. What is the most likely diagnosis?

A. Hyperesthesia of dental hard tissuesB. Enamel erosionC. Enamel necrosisD. Enamel hypoplasiaE. Pathological abrasion of dental hardtissues

6. After removal of dental plaque an 18-year-old patient underwent preventiveexamination. It revealed painless chalkyspots in the precervical region on thevestibular surface of the 22 and 41 teeth.Result of enamel resistance test is 7. Whatmorphological changes are typical for thisdisease?

A. Subsurface enamel demineralizationB. Changes in the mantle dentineC. Damage of dentinoenamel junctionD. Superficial enamel demineralizationE. Degeneratic changes of odontoblasts

7. A 49-year-old patient complains ofconstant mandibular pain irradiatingto the ear, soft tissue edema, bodytemperature rise up to 39, 0oC. Objecti-vely: there is an edema in the submandi-bular region, the skin has not changed incolour. Alveolar mucosa around the 45,46 teeth is hyperemic and edematic on theoral and vestibular side. Crowns of the45, 46 teeth are completely destroyed, theteeth are mobile. Vincent’s symptom ispresent. What is the most likely diagnosis?

A. Acute mandibular osteomyelitisB. Abscess of submandibular regionC. Exacerbation of chronic mandibularosteomyelitisD. Acute serous periostitis of mandibleE. Acute purulent periostitis of mandible

8. An 8-year-old boy complains of havi-ng toothache during eating. Objectively:approximal surface of the 55 tooth hasa deep carious cavity communicating wi-th the tooth cavity. Probing causes acutepain, there is bleeding, percussion is pai-

Page 2: Krok 2 - 2012 Question Paper (Stomatology)

Krok 2 Stomatology 2012 2

nless. What is the most likely diagnosis?

A. Chronic fibrous pulpitisB. Chronic hypertrophic pulpitisC. Chronic gangrenous pulpitisD. Chronic granulating periodontitisE. Chronic fibrous periodontitis

9. A 36-year-old patient complains ofpain under the dental bridge. After itsremoval the patient has been found tohave an ulcer 0,3x0,5 cm large on thealveolar process. The ulcer is slightly pai-nful and soft, the surrounding mucosa ishyperaemic, submandibular lymph nodesare not enlarged. What is a provisional di-agnosis?

A. Decubital ulcerB. Trophic ulcerC. Sutton aphthaD. Cancerous ulcerE. Tuberculous ulcer

10. A 45-year-old female patient complai-ns of worsened fixation and frequentbreakages of her partial removable lami-nar denture for the lower jaw, whichshe has used for 5 years. Objectively:alveolar process in edentulous areas isconsiderably atrophied, the denture keepsbalance. What is the most likely cause ofworsened fixation and frequent breakagesof the denture in this case?

A. Atrophy of the alveolar processB. Wear of artificial teethC. Improper keeping of the dentureD. Using the denture during sleepE. Eating solid food

11. A 42-year-old female patient complai-ns of tooth mobility, difficult masticati-on. Objectively: face configuration isunchanged. The 35, 36, 38, 44, 46, 48 teethare missing. The 31, 32, 41, 42 teeth have1-2 grade mobility. What is the most effici-ent denture construction in this case?

A. Whole-piece splint with vestibularclowlike hooksB. Whole-piece splint with Roach claspsand a multijoint claspC. Mamlock splintD. Removable plastic splintE. Interdental splint

12. A 42-year-old patient complains ofpain in the submaxillary and sublingualareas that is getting worse during eating,body temperature rise up to 37, 6oC. Hehas been suffering from this for 2 months.Objectively: infiltration along the right

sublingual torus, hyperemia, soft tissueedema, acute pain during palpation. Theduct of the right submandubular salivarygland excretes turbid saliva mixed withpus. What is the most likely diagnosis?

A. Exacerbation of salivolithiasisB. Acute purulent lymphadenitis ofsubmaxillary areaC. Adenophlegmon of submaxillary areaD. Abscess of maxillolingual grooveE. Retention cyst of sublingual salivarygland

13. A 4-year-old girl presents with bodytemperature rise, aggravation of generalcondition. The symptoms has beenobserved for 3 days. Objectively: generalcondition is grave, body temperature is38, 6oC, the girl is anxious and pale. Shepresents also with halitosis, hyperaemiaand edema of gingival mucous membranein the region of the 83, 84, 85 teeth onboth sides from the alveolar process.The mentioned teeth are mobile, theirpercussion causes acute pain; the 84 toothis filled. What is the most likely diagnosis?

A. Acute odontogenous mandibularosteomyelitis beginning from the 84 toothB. Acute sialoadenitis of submandibularsalivary glandC. Exacerbation of chronic periodontitis ofthe 84 toothD. Suppuration of the radiculodentalmandibular cyst beginning from the 84toothE. Acute odontogenous mandibular peri-ostitis beginning from the 84 tooth

14. A 48-year-old female patient hadbeen delivered to a hospital with a cheekwound and severe headache, nausea, di-zziness. It is known from the history thatthe patient got a trauma as a result ofa fall. After examination she was di-agnosed with a contused lacerated woundof cheek, closed craniocerebral injury,brain concussion. This trauma can bequalified as:

A. Concominant traumaB. Isolated traumaC. Combined traumaD. PolytraumaE. Single

15. A 26-year-old patient needs prostheti-cs. Objectively: crown of the 16 tooth isdestroyed by 1/3. It is planned to restoreits anatomical shape with a metal inlay.What is the first stage of making a cavityfor the inlay?

Page 3: Krok 2 - 2012 Question Paper (Stomatology)

Krok 2 Stomatology 2012 3

A. Removal of affected tissuesB. Making additional cavitiesC. Making a bevelD. Making the cavity floorE. Making the cavity walls

16. An 8-year-old boy complains ofimproper arrangement of teeth. Exami-nation at an orthodontic clinic revealedbroad, tight, low-attached upper lipfrenulum. Broad frenulum and its lowattachment may cause:

A. DiastemaB. Shortening of the upper dentitionC. Elongation of the upper dentitionD. Narrowing of the upper dentitionE. Protrusion of the upper front teeth

17. During extraction of the 47 tooth itsdistal root was broken halfway along itslength. What tool should be chosen forextraction of the residual root fragments?

A. Left angled elevatorB. Broad-beaked forcepsC. Close-beaked forcepsD. Right angled elevatorE. Straight elevator

18. A 22-year-old student complains of gi-ngival haemorrhage during tooth brushi-ng. Objectively: hyperaemia and edema ofmarginal gingiva of the front teeth on bothjaws, periodontal pockets are absent. X-ray reveals no pathological changes. Whatis the most likely diagnosis?

A. Chronic catarrhal gingivitisB. Localized periodontitisC. Ulcerative gingivitisD. Hypertrophic gingivitisE. Generalized periodontitis

19. A 37-year-old male patient complai-ns about pain of the 46 tooth duringfood intake, especially hot food, offensivebreath when he sucks his tooth. Objecti-vely: the face is symmetrical, masticatorysurface of the 48 tooth has a deep cari-ous cavity communicating with the dentalcavity. X-ray picture shows widening ofperiodontal fissure at the root apex ofthe 46 tooth. What is the most likely di-agnosis?

A. Chronic gangrenous pulpitisB. Exacerbation of chronic periodontitisC. Exacerbation of chronic pulpitisD. Chronic fibrous periodontitisE. Chronic fibrous pulpitis

20. A 24-year-old male complains ofpain, bad breath, body temperature ri-se up to 38, 0oC. Objectively: the pati-ent is pale, adynamic. Regional lymphnodes are enlarged and painful. Gums areedematic, hyperaemic, ulcerated, coveredwith necrotic plaque. There is an excessivebuildup of calculus. What additional studyshould be done in the first place?

A. Complete blood countB. Blood sugar testC. Microscopy of gingival plaqueD. Test for HIV infectionE. X-ray of jaws

21. A 46-year-old patient complains ofspontaneous pain in the 36 tooth. Itscrown is decayed. The patient presentedwith edema of soft tissues adjacent to themandible. Body temperature rose up to39, 0oC, the patient has chill. Examinationof the oral cavity revealed a pronouncededema of the left mandibular alveolarprocess on the vestibular and oral side(acute periostitis); the 35, 36, 37 teethare mobile, subgingival pockets containpurulent exudate; their percussion causesacute pain. Positive Vincent’s symptomis present. What is the presumptive di-agnosis?

A. Acute odontogenic osteomyelitis ofmandible from the 36 toothB. Exacerbation of chronic periodontitisfrom the 36 toothC. Acute purulent periostitis of mandiblefrom the 36 toothD. Exacerbation of generalized periodonti-tisE. Exacerbation of chronic mandibularosteomyelitis

22. An 11-year-old girl complains about gi-ngival haemorrhage during tooth brushi-ng and eating. She has been sufferi-ng from this for a year. Gum of bothupper and lower jaws is edematic andcongestively hyperemic. Hygienic state oforal cavity is unsatisfactory. Bite is edge-to-edge. Roentgenological examinationof periodontium revealed no pathologi-cal changes. What is the provisional di-agnosis?

A. Chronic catarrhal gingivitisB. Acute catarrhal gingivitisC. Generalized periodontitisD. Localized periodontitisE. Hypertrophic gingivitis

23. Preventive examination of a 4,5-year-

Page 4: Krok 2 - 2012 Question Paper (Stomatology)

Krok 2 Stomatology 2012 4

old child revealed some hidden cavitieson the contact surfaces of the 54 and 55teeth. After removal of the overhangingedges of the enamel the softened dentincould be easily removed within the mantledentin. Select the optimal material for apermanent filling:

A. Compomer materialB. Composite materialC. Silicate cementD. Silicophosphate cementE. Polycarboxylate cement

24. A 54-year-old woman complains aboutwear of the 35 and 36 teeth and paincaused by thermal and chemical stimuli.Objectively: crowns of the 35 and 36 teethare worn by 1/3 of their height (horizontaltype), the 24, 25 26 teeth have fullcastcrowns. What crowns should be made forthe 35 and 36 teeth?

A. FullcastB. Combined Belkin’sC. Metal stampedD. PlasticE. Metal-ceramic

25. An 18-year-old patient complainsabout body temperature rise, weakness,pain induced by eating and deglutition.Objectively: mucous membrane of theoral cavity is erythematic with multi-ple petechia. Pharynx is hyperaemic.Regional lymph nodes are enlarged,mobile, painless. In blood: leukocytosis,monocytosis, atypic mononuclear cells,ESR is 30 mm/h. What is the leadingfactor of disease development?

A. Viral infectionB. Bacterial infectionC. Autoimmune disordersD. Immediate allergyE. Delayed allergy

26. An 18-year-old student needsprosthetic metal-ceramic denture for the11, 21 teeth. There are no contraindicati-ons for the use of such construction. Whatis the most appropriate material for taki-ng impressions?

A. SielastB. StomalginC. OrthocorD. StensE. Repin

27. A soldier was delivered to the regimentmedical station with dislocation asphyxiacaused by a gunshot wound. What actions

should be taken for asphyxia suppression?

A. Pul the tongue forwards and sew itthroughB. Intubation of tracheaC. Tracheostoma establishmentD. Artificial airwayE. Artificial pulmonary ventilation

28. A week ago an 18-year-old girlcomplained of pain in the 22 tooth whi-ch was treated and filled several yearsago. Over the past two days the pain gotworse. Objectively: the 22 tooth is filled,percussion is painful, mucous membraneis hyperaemic and edematic. Spot-filmroentgenograph of the 22 tooth showsan ill-defined pariapical bone rarefaction0,4x0,5 cm large. What is the most likelydiagnosis?

A. Exacerbation of chronic periodontitisof the 22 toothB. Suppuration of the radicular cystC. Acute odontogenic osteomyelitisD. Acute purulent periodontitis of the 22toothE. Acute maxillary periostitis

29. A 22-year-old patient complains of apainful swelling in the right parotid gland.A week earlier the patient got a cheekabrasion which healed under the purulentcrust. Over the past two days the patienthad observed progressing pain and feverup to 38, 6oC. Objectively: there is a softtissue edema in the right parotid region,the skin is slightly strained but has notchanged in colour. There is a dense painfulinfiltration 2,5x3,5 cm large, the skin overit exhibits limited mobility. The mouth canbe fully opened, the mucous membranearound the orifice of the salivary duct isunchanged, the saliva is transparent. Whatis the most likely diagnosis?

A. Acute lymphadenitisB. Exacerbation of chronic parotitisC. Abscess of the parotid-masticatoryregionD. Acute non-epidemic parotitisE. Epidemic parotitis

30. A 33-year-old female patient has beenadmitted to the maxillofacial departmentwith complaints of pain and edema ofthe right submandibular region, bodytemperature rise up to 39, 5oC. Objecti-vely: the patient has asymmetric facebecause of soft tissue edema of the rightsubmandibular region, palpation revealsa dense infiltration, the skin over it ishyperemic, does not make a fold. The 46

Page 5: Krok 2 - 2012 Question Paper (Stomatology)

Krok 2 Stomatology 2012 5

tooth has a deep carious cavity. What isthe most likely diagnosis?

A. Submandibular phlegmon on the rightB. Acute submandibular sialoadenitisC. Acute purulent periostitis of mandibleD. Acute purulent submandibularlymphadenitisE. Acute right-sided osteomyelitis ofmandible

31. Analysis of the contents of periodontalpockets revealed a significant contami-nation with Candida yeast fungi. Whichof the following drugs should be used forinstillations?

A. ClotrimazoleB. TrichopolC. DioxydinD. Diclofenac sodiumE. Tinidazole

32. A 25-year-old patient consulteda doctor about massive gingivalhaemorrhages, dry mouth, mobility andshifting of teeth, purulent discharges fromthe gums, bad breath. According to thepatient, these presentations turned upabout 2 months ago. Before the diagnosiscan be made, the following tests should bedone in the first place:

A. Blood sugar testB. Immunological studiesC. Allergological testsD. Acute-phase reactants testsE. Serologic studies

33. A 59-year-old patient has a bilateralfracture of mandible in the region of the44, 34 teeth, other masticatory teeth aremissing, toothless fragments are not di-splaced but mobile. Which orthosis shouldbe used for the immobilization of bonefragments?

A. Vankevich splint with orthodonticelasticsB. Rudko’s apparatusC. Limberg’s splintD. Petrosov’s apparatusE. Zbarzh apparatus

34. A 20-year-old patient has asymmetricface due to an upper lip edema on the left.The skin over it is bluish, there are positi-ve symptoms of "compression"and "fillingin". What is the most likely diagnosis?

A. Cavernous hemangiomaB. Branched hemangiomaC. Capillary hemangiomaD. LymphangiomaE. Lentigo

35. A 3-year-old girl complains of painand swelling in the region of the decayed51, 52 teeth, body temperature rise upto 37, 5 − 37, 9oC. Objectively: the faceis asymmetric because of a swelling inthe upper lip region and right infraorbi-tal region. The crown of the 51 tooth iscompletely decayed. Mucous membranein the region of the 52, 51, 1 teeth isedematic, mucogingival fold is smoothed,palpation provokes pain, mobility of I-IIgrade of the 51, 52 teeth is also present.What is the most likely diagnosis?

A. Acute purulent odontogenic maxillaryperiostitisB. Acute albuminous odontogenic maxi-llary periostitisC. Acute odontogenic maxillaryosteomyelitisD. Odontogenic abscess of infraorbitalregionE. Exacerbation of chronic periodontitisof the 51 tooth

36. A 31-year-old male patient complai-ns of dryness and burning of tongueback that appeared for about a weekago and get worse when he eats irritati-ng food. The patient has a history ofrecent pneumonia. He had been treatedin the in-patient hospital for 2 weeks, thetreatment program included antibiotics.Now he doesn’t take any drugs. Objecti-vely: mucous membrane of the oral cavityis hyperemic, dry, glossy. Tongue back andpalate have greyish-white plicae that canbe easily removed. Threads of saliva trailbehind the spatula. What is the most likelydiagnosis?

A. Acute pseudomembranous candidiasisB. Chronic hyperplastic candidiasisC. Acute atrophic candidiasisD. Medicamental stomatitisE. Chronic atrophic candidiasis

37. During the examinations and everydayorthopaedic manipulations a dentist usesa dental mirror. What is the way of dentalmirrors sterilization?

Page 6: Krok 2 - 2012 Question Paper (Stomatology)

Krok 2 Stomatology 2012 6

A. In the triple solution for 30 minutesB. In the 0,5% ethyl chloride solution for20 minutesC. In the 6% hydrogen peroxide solutionfor 6 hoursD. In the dry heat sterilizer at 180oC for 10minutesE. In the 0,01% chloramine solution for 10minutes

38. A 65-year-old patient complains aboutpartially missing teeth on his upper jaw,difficult mastication, rhinolalia. Objecti-vely: the 18, 16, 15, 11, 23, 28, 35, 38, 48,47 teeth are missing; there is postoperati-ve midline defect of hard palate. It wasdecided to make a clasp dental prosthesiswith obturating part. The obturating partshould be placed on the following elementof the clasp dental prosthesis:

A. On the archB. On the saddleC. On the artificial teethD. On the baseE. On the clasps

39. Examination of an 11-year-old boyrevealed thickened, somewhat cyanotic,dense gingival margin overlapping thecrowns of all teeth by 1/2 of their height.Fedorov-Volodkina oral hygiene index is2,6, PMA index is 20%. X-ray pictureshows no pathological changes of peri-odontium. The child has a 2-year historyof neuropsychiatric treatment for epilepsy.Make a provisional diagnosis:

A. Chronic hypertrophic gingivitisB. Chronic catarrhal gingivitisC. Localized periodontitisD. Acute catarrhal gingivitisE. Generalized periodontitis

40. A 39-year-old patient complains of acosmetic defect, hypersensitivity of the12, 11, 22 teeth. Objectively: vestibularsurface of these teeth has oval defects wi-th smooth shiny walls. Probing causes nopain, there is a pain reaction to cold sti-muli. The defects can be stained with 5%tincture of iodine. What is the most likelydiagnosis?

A. Enamel erosionB. Superficial cariesC. Wedge-shaped defectD. Systemic hypoplasiaE. Fluorosis, erosive form

41. A 70-year-old patient complains ofpain in the lower jaw region on the right.He has been using complete removable

dentures for the upper and lower jawfor 12 years. He is smoker. Objecti-vely: the right retromolar region exhi-bits a 1,5х1,2 cm large proliferation ofmucous membrane in form of cauliflower,here and there there are dense fissure-like ulcers. The surrounding mucousmembrane is cyanotic, infiltrated. Whenthe teeth are closed, the mentionedformation contacts with the posterioredges of the dentures. What is the provisi-onal diagnosis:

A. Cancer of the mucous membraneB. LeukoplakiaC. Decubital ulcerD. Hypertrophic gingivitisE. Papillomatosis

42. A 40-year-old patient complains ofdiscoloration of the vermilion border ofthe lower lip that he noticed about 4months ago. Objectively: in the center ofthe vermilion border of the lower lip thereis an irregular homogeneous grayish-white area 1x1,5 cm large that doesn’t riseabove the vermilion border and has di-stinct outlines. Palpation of this area ispainless, the surrounding tissues are notchanged. The film cannot be removedwhen scraped. The 31, 32, 41, 42 teeth aremissing. What is the most likely diagnosis?

A. LeukoplakiaB. Lupus erythematosusC. Lichen ruber planusD. Candidous cheilitisE. Premalignant circumscribedhyperkeratosis

43. A 35-year-old patient complainsabout itch, burning and edema of lips.He has been suffering from this for aweek. Objectively: reddening of vermi-lion border and skin, especially in theregion of mouth corners, there are alsovesicles, crusts, small cracks along wi-th erythematous affection of vermilionborder. What is the most likely diagnosis?

A. Acute eczematous cheilitisB. Multiform exudative erythemaC. Acute herpetic cheilitisD. Allergic contact cheilitisE. Exudative form of exfoliative cheilitis

44. A 45-year-old man complains about li-quid outpouring from his nose, inabilityto blow his nose, inflated cheeks. Objecti-vely: there is a perforating defect (1х1,5cm) of alveolar process at a level of theextracted 26th tooth in the lateral part

Page 7: Krok 2 - 2012 Question Paper (Stomatology)

Krok 2 Stomatology 2012 7

of his upper jaw. Air inhalation throughthe nose with held nostrils is accompani-ed by generation of bubbles in the areaof perforation. What denture constructi-on should be recommended?

A. Minor saddle denture with clasp fixati-onB. Clasp denture with obturating partC. Common partial removable dentureD. Common dental bridgeE. Protective palatal bars

45. Preventive examination of an 8-year-old boy revealed some lusterless chalk-like spots on the vestibular surface of the11 and 21 teeth, which are localised in theprecervical region. Subjective complaintsare absent. What is the most likely di-agnosis?

A. Acute initial cariesB. White-spotted fluorosisC. Local enamel hypoplasiaD. Acute superficial cariesE. Chronic initial caries

46. A 44-year-old male patient complainsof fatigue and headache, limb numbness,dry mouth, burning and pain in thetongue. Objectively: skin and oral mucosaare pale. There are painful cracks in thecorners of mouth. Dorsum of tongueis smooth, glossy, with bright red stri-ae. In blood: Hb- 70 g/l, RBCs - 1, 5 ·1012/l, color index - 1,6, leukopenia,thrombocytopenia, lymphocytosis. Whatis the most likely diagnosis?

A. Addison-Biermer anemiaB. Chronic posthaemorrhagic anemiaC. Late chlorosisD. Iron deficiency anemiaE. Aplastic anemia

47. A 25-year-old patient presents wi-th fluorosis of vestibular surfaces of theupper incisors. Which of the followingconstructions will provide the maximumaesthetic result?

A. VeneersB. Plastic crownC. Ceramic crownD. Combined Kurylenko crownE. Metal-plastic crown

48. A 27-year-old patient complains aboutacute pain in the region of the 34 tooththat is getting worse when biting downon food. Roentgenographical surveyrevealed an ill-defined zone of bone ti-ssue destruction in the periapical region

of root of the 34 tooth. What is the mostlikely diagnosis?

A. Exacerbation of chronic granulatingperiodontitisB. Acute purulent pulpitis complicated byperiodontitisC. Exacarbation of chronic pulpitisD. Exacerbation of chronic granulomatousperiodontitisE. Acute serous periodontitis

49. A 48-year-old patient complains ofsubfebrile temperature and a growingulcer on the gingival mucosa around themolars; looseness of teeth in the affectedarea, cough. Objectively: gingival mucosain the region of the lower left molars hastwo superficial, extremely painful ulcerswith undermined edges. The ulcers floor isyellowish, granular, covered with yellowi-sh, and sometimes pink granulations. Theulcers are surrounded by the tubercles.Dental cervices are exposed, there isa pathological tooth mobility. Regionallymph nodes are enlarged and make densematted together groups. What is the mostlikely diagnosis?

A. TuberculosisB. SyphilisC. Acute aphthous stomatitisD. Infectious mononucleosisE. Decubital ulcer

50. A 56-year-old man complains of painin the left parotidomasticatory region,progressing face asymmetry that was noti-ced a month ago. Objectively: left-sidedparesis of mimic muscles. To the fore ofearflap there is an ill-defined infiltration,the skin above it is tense and cyanotic;left lymph nodes are enlarged. Opening ofmouth is limited down to 2,5 cm. The leftparotid duct doesn’t excrete saliva. Whatis the most likely diagnosis?

A. AdenocarcinomaB. Mixed tumourC. Cyst of the glandD. Glandular tuberculosisE. Chronic lymphadenitis

51. A 4-year-old girl complains ofspontaneous acute toothache in the upperjaw on the right increasing during eating.The pain arose two days ago. Objectively:there is a deep carious cavity filled wi-th softened dentin in the 55 tooth. Thereis no intercommunication with the toothcavity. Probing of the cavity floor is sli-ghtly painful. Percussion is painful. Whatis the presumptive diagnosis?

Page 8: Krok 2 - 2012 Question Paper (Stomatology)

Krok 2 Stomatology 2012 8

A. Acute pulpitis complicated by peri-odontitisB. Acute purulent pulpitisC. Acute generalized (serous) pulpitisD. Exacerbation of chronic gangrenouspulpitisE. Acute purulent periodontitis

52. A 35-year-old man complains of short-term pain caused by thermal stimuli in the46 and 47 teeth. Objectively: masticatorysurfaces of the 46, 47 teeth are intact,approximal surfaces could not be exami-ned because of a very close arrangementof teeth. What methods of study canbe used in this case if you suspect thepresence of hidden cavities?

A. X-rayB. Vital stainingC. Fluorescent studyD. Measuring electrical resistanceE. Electro-odontometry

53. A 52-year-old man undergoestreatment of the 36 tooth for chronicperiodontitis. Radiography revealedconvoluted medial root canals; anirregular-shaped focus of destruction ofbone tissue 0,2x0,2 cm large in the regi-on of the medial root apex. Which of thefollowing drugs is optimal for intracanalelectrophoresis?

A. 10% solution of potassium iodideB. 1% solution of decamethoxinC. 1% solution of novocaineD. 3% solution of copper sulphateE. 0,1% solution of trypsin

54. A 44-year-old patient consulted asurgeon about constant acute pain in theupper jaw region on the left that is getti-ng worse during teeth joining. The painwas noted 3 days ago. Objectively: theface is symmetrical, mouth opening isunlimited. The crown for the 26 toothis half-destroyed. Probing of the cari-ous cavity is painless. Percussion of the26 tooth provokes acute pain. Mucousmembrane of the alveolar process isedematic, hyperaemic at the level ofthe 26 tooth. The 26 tooth had beentreated before. What is your provisionaldiagnosis?

A. Exacerbation of chronic periodontitisof the 26 toothB. Acute purulent periodontitis of the 26toothC. Acute pulpitis of the 26 toothD. Acute purulent periostitis of upper jawof the 26 tooth on the leftE. Periodontitis of the 26, 27, 28 teeth

55. A 22-year-old medical studentcomplains of the changed colour of hiscentral upper incisor on the right. Twoyears ago the tooth was gray. Objectively:the 2 tooth is filled, changed in colour,stable, percussion is painless. The pati-ent has closed bite. What is the absolutecontraindication to making porcelaincrowns for this patient?

A. Closed biteB. Defects of the anterior teeth that cannotbe restored by fillingsC. Enamel hypoplasia with a change oftooth shape and colourD. Abnormal tooth colourE. Defects of pulpless teeth that can notbe restored by inlays

56. A 18-year-old patient complains ofacute spontaneous toothache irradiati-ng to the right eye and temporal region.Objectively: there is a deep carious cavi-ty in the 27 tooth within circumpulpardentin. Dentin is light, softened. Probi-ng of the cavity floor and cold test causeacute pain. What is the most likely di-agnosis?

A. Acute diffuse pulpitisB. Acute purulent periodontitisC. Exacerbation of chronic pulpitisD. Acute serous periodontitisE. Acute purulent pulpitis

57. A 43-year-old patient was found tohave no crown of the 13 tooth. The rootprotrudes above the gum by 0,5-1 mm.The apical part is filled. History: coronalpart of the tooth broke off three weeksago. Prior to this the tooth had beentreated for chronic periodontitis. Percussi-on is painless. The root edges are made byhard moist mucous membrane of pale pi-nk colour. Specify the dentist’s tactics:

A. Fabricate a crown-root inlay and metal-ceramic crownB. Remove the tooth rootC. Fabricate a simple pivot crownD. Restore the tooth by means of ananchor and photopolymer materialE. Fabricate a pivot Richmond crown

Page 9: Krok 2 - 2012 Question Paper (Stomatology)

Krok 2 Stomatology 2012 9

58. A 40-year-old patient with mandiblefracture consulted a doctor 3 weeks afterimmobilization of fragments because ofpain and body temperature rise. Objecti-vely: a slight swelling in the mentalregion, mucous membrane of alveolarprocess in the area of the 21|12 teeth ishyperaemic, edematic, palpatory painful.Overtooth splint on 54321|12345 teethis in satisfactory condition, no occlusionabnormalities were detected. The patientwas diagnosed with acute purulent peri-ostitis of mandible. What surgical action isindicated?

A. Lancing of abscess to the boneB. Intraoral novocaine blockC. Removal and replacement of theovertooth splint by a new oneD. Trepanation of the 21 and 12 teethE. Surveillance of patient

59. A 32-year-old patient complains ofacute spontaneous attacks of pain in the14 tooth. The pain lasts for 10-20 minutesand occurs every 2-3 hours. Carious cavi-ty in the 14 tooth is filled with softeneddentin. Probing of the cavity floor is pai-nful at one point. Cold stimulus causespain. What is the most likely diagnosis?

A. Acute localized pulpitisB. Acute deep cariesC. Hyperemia of the pulpD. Exacerbation of chronic pulpitisE. Acute diffuse pulpitis

60. A 32-year-old patient complains of thelong-term dull toothache caused by hotfood. The toothache appeared a monthago. Objectively: the 26 tooth has changedin colour, on the masticatory surface thereis a deep carious cavity communicatingwith the tooth cavity. Superficial probingof pulp is painless, deep probing is painful.Electro-odontodiagnostics results: 85 µA.What is the most likely diagnosis?

A. Chronic gangrenous pulpitisB. Chronic hypertrophic pulpitisC. Chronic fibrous pulpitisD. Chronic fibrous periodontitisE. Chronic concrementous pulpitis

61. A 27-year-old patient complains of thelong-term pain in the 22 tooth causedby cold and hot food, as well as ofspontaneous pain lasting for 30 minutesand occurring 3-4 times per day, gettingworse at night. Pain arose 3 days ago afterpreparation of the tooth for the acryliccrown. Objectively: the 22 tooth is intact,

the thermal test causes acute long-lastingpain, percussion is painless. What is theoptimal treatment tactics?

A. Vital pulp extirpationB. Application of fluorine lacquerC. Biological treatment of pulpitisD. Devital pulp extirpationE. Vital pulp amputation

62. A 5-year-old child has been di-agnosed with congenital completenonclosure of soft and hard palate.What type of anaesthesia is indicated foruranostaphyloplasty?

A. Nasotracheal narcosisB. Mask narcosisC. Intravenous narcosisD. Orotracheal narcosisE. Endotracheal narcosis throughtracheostome

63. A 24-year-old patient complains ofaching pain in the 11 tooth that is getti-ng worse on biting. Two days ago thetooth was filled for pulpitis. Objectively:the 11 tooth is filled. The thermal testcauses no pain, vertical percussion is sli-ghtly painful. X-ray picture of the 11 toothshows that the endodontic filling is 1 mmabove the root apex. Which of the followi-ng methods will be most effective for eli-minating this complication?

A. FluctuorizationB. Ultrahigh frequency therapyC. Relaxing incisionD. Submucous injection of 1% solution ofhydrocortisoneE. Analgetics

64. It is planned to make a metal-ceramiccrown supported by stump inlay for the23 tooth. Objectively: the crown of the23 tooth is decayed down to the gingivaledge. Root canal is filled to the top. Thedentist made a wax stump model with apin, cast it in metal, fitted it to the tooth,fixed it by means of visphat-cement andgot a working plaster impression. At whatstage did he make an error?

A. Impression takingB. Wax construction makingC. CastingD. Stump fittingE. Stump fixing

65. A 23-year-old military servant needsorthopaedic treatment at a specializedhospital. He was diagnosed with false jointof mandible in its frontal part. The teeth

Page 10: Krok 2 - 2012 Question Paper (Stomatology)

Krok 2 Stomatology 2012 10

are intact, stable, in threes on each side.Orthopaedic treatment by means of a bri-dge denture will be possible only if the jawdefect is no more than:

A. 1 cmB. 2 cmC. 3 cmD. 3,5 cmE. 4 cm

66. According to the mother, a 5-year-old child complains about pain duringswallowing, weakness, body temperaturerise upt to 39, 5oC, swelling of submentallymph nodes. Objectively: the child’scondition is grave, body temperature is38, 8oC. Mucous membrane of oral cavityis brightly hyperaemic and edematic withhaemorrhages and ulcerations. Pharynx isbrightly hyperemic, lacunae are enlargedand have necrosis areas. Regional, cervi-cal, occipital lymph nodes are painful,enlarged and dense. What is the most li-kely diagnosis?

A. Infectious mononucleosisB. Acute herpetic stomatitisC. Necrotizing ulcerative gingivostomatitisD. Herpetic anginaE. Lacunar tonsillitis

67. A patient is 48 year old, accordingto the results of clinicoroentgenologicalexamination it is indicated to removethe 26 tooth because of acute conditionof chronic granulomatous periodontitis.What kind of conduction anesthesia isindicated for this operation?

A. Tuberal and palatinalB. TorusC. Infraorbital and incisiveD. PlexusE. Infraorbital and palatinal

68. Examination of an 8-year-old childrevealed irregular white spots on the vesti-bular surface in the precervical regionof the of 11 and 12 teeth. The spots aresmooth and stainable. What is the mostlikely diagnosis?

A. Focal demineralizationB. Enamel hypoplasiaC. Superficial cariesD. Enamel erosionE. Fournier’s teeth

69. A 45-year-old patient complains ofmissing teeth in the lower jaw on the ri-ght. Objectively: the 46, 45, 38, 48 teethare missing. The 47 tooth is mobile (1

grade mobility), the crown of the 44 toothis destroyed by 1/2. What orthopedicconstruction should be recommended forrestoration of masticatory efficiency andprevention of periodontal overload of the47, 44 teeth?

A. Arch denture with clasp fixation for the47, 44, 34 teethB. Soldered bridge supported by the 48, 44teethC. Metal-ceramic bridge supported by the47, 46 teethD. Bridge supported by the 47 toothE. Small saddle denture with clasp fixationfor the 47, 44 teeth

70. A 57-year-old patient complains oftooth mobility, inability to eat. Objecti-vely: the lower 35, 36, 37, 38, 44, 45, 46and 48 teeth are missing; the 31, 32, 33, 34,41, 42, 43, 47 teeth exhibit II grade mobi-lity, their clinical crowns are low, toothequator is not pronounced. What is theoptimal denture construction in this case?

A. Removable whole-piece splintB. Removable partial dentureC. Kurliandsky splint barD. Removable Bynin splintE. Removable splint with vestibulo-oralclasp

71. A patient working as a bricklayercomplains of itching, burning, soreness oflips that show up only in the summer peri-od. He has been ill for 3 years. Objecti-vely: vermilion border of the lower lip ishyperemic, edematic, covered with bli-sters and painful erosions 2 mm in di-ameter, crusts, cracks. What is the mostlikely diagnosis?

A. Actinic cheilitis, exudative formB. Contact allergic cheilitisC. Meteorological cheilitisD. Eczematous cheilitis, exudative formE. Exfoliative cheilitis, exudative form

72. A 60-year-old patient complains ofpain in the masticatory muscles andtemporomandibular joints. The painoccurs when she uses complete removabledentures fabricated a month ago. Objecti-vely: the lower third of the face iselongated, the lips can be closed withdifficulty, smiling causes exposure of thedenture base, the articulation is impaired.A mistake must have been made at thefollowing stage of denture fabrication:

Page 11: Krok 2 - 2012 Question Paper (Stomatology)

Krok 2 Stomatology 2012 11

A. Determination and fixation of thecentral occlusionB. Taking anatomic impressionsC. Taking functional impressionsD. Testing the denture constructionE. Fitting the dentures and their correction

73. A patient consulted a dentist about acosmetic defect in the cervical region ofthe upper and lower canines. Various sti-muli cause no pain. Objectively: there areV-shaped defects on the vestibular surfacein the cervical area of the upper and lowercanines. Their surface is smooth, shiny,hard. The reaction to probing and cold sti-muli is absent. What treatment should beadministered?

A. Filling of the defectsB. Metal crownsC. Applications with 10% solution ofcalcium gluconateD. Application of fluorine lacquerE. Medical intervention is not necessary

74. A 57-year-old patient complains aboutdryness and burning of the lateral surfaceof her tongue. These sensations disappearduring eating. She noted such sensationsthree months ago. She has a history ofgastritis with reduced secretory function.Objectively: mucous membrane of tongueand oral cavity has no peculiarities. Theback of tongue has thin white coating.Regional lymph nodes are unpalpable.Oral cavity is sanitized. What is the mostlikely diagnosis?

A. GlossodyniaB. Lingual nerve neuritisC. CandidiasisD. Desquamative glossitisE. Hunter-Moeller glossitis

75. A 65-year-old patient needs completeremovable dentures for both jaws. At thestage of "testing the denture constructi-on"the doctor checks the pronunciationof sounds "S"and "Z". Which method ofnormalization of speech functions shouldbe applied in this case?

A. Phonetic testsB. GraphicC. MyogymnasticsD. SpectrographicE. Acoustic

76. An 18-year-old man complains ofthe 14, 13, 12, 23, 24 teeth being sensiti-ve to sweet and sour food. Examinationrevealed some isolated ill-defined chalkyspots in the precervical region. What is

the most informative method of study thatallows to confirm the diagnosis?

A. Vital stainingB. X-rayC. ProbingD. Electro-odontometryE. Thermometry

77. Parents of a 12-year-old child complainof white patches on the upper front teeth,which appeared six months ago. Objecti-vely: chalk-like patches on the vestibularsurfaces in the precervical region of the13, 12, 11, 21, 22, 23 teeth. Their enamel isdull, pliable and rough on probing. Thereis a history of short-term pain caused bychemical stimuli. What is your provisionaldiagnosis?

A. Acute initial cariesB. Chronic initial cariesC. Acute superficial cariesD. Systemic enamel hypoplasiaE. Dental fluorosis

78. A 32-year-old female patient needsdental prosthetics. After the objecti-ve examination it was decided to useceramic-metal crown. What impressi-on material should be used for takingimpressions for this construction?

A. StomaflexB. PlasterC. StomalginD. StensE. Orthocor

79. A 55-year-old patient has a painless,tuberous, cyanotic pedunculated formati-on 2х1х1,5 cm large that appeared on thesite of the extracted 46 tooth. Openingof mouth is not limited. Intra-oral X-raypicture of alveolar process in the regionof the removed 46 tooth shows a focus ofbone tissue destruction. What is the mostlikely diagnosis?

A. Giant-cell epulisB. Hard odontoma of mandibleC. Hypertrophic gingivitisD. Papilloma of mucous membrane in thearea of the extracted 46th toothE. Ameloblastoma of mandible

80. A 52-year-old patient presented tothe prosthetic dentistry clinic complaini-ng of missing upper jaw teeth and difficultmastication. The patient has an indicati-on for a partial laminar denture for theupper jaw with retaining clamps on the14, 23 teeth. What kind of denture stabili-

Page 12: Krok 2 - 2012 Question Paper (Stomatology)

Krok 2 Stomatology 2012 12

zation will enable such positioning of theclasps?

A. TransversalB. SagittalC. DiagonalD. Sagitally-transversalE. Diagonally-sagittal

81. A 56-year-old patient presents to adental clinic. He has an indication for theextraction of the 22 tooth. Examinationrevealed periosteal abscess in the projecti-on of the 21, 22, 23 teeth, flattening ofthe mucogingival junction. What kind ofperipheral conduction anaesthesia is mostadvisable?

A. Extraoral infraorbital anaesthesia +incisor anaesthesiaB. Intraoral infraorbital anaesthesia +incisor anaesthesiaC. Intraoral infraorbital anaesthesia +palatinal anaesthesiaD. Plexus anaesthesia in the projection ofthe 22 tooth + incisor anaesthesiaE. Plexus anaesthesia in the projection ofthe 22 tooth + palatinal anaesthesia

82. A 33-year-old patient complai-ns about pain, gingival haemorrhage,halitosis, body temperature rise upto 37, 8oC. Objectively: gums areapparently hyperaemic, edematic, bleedeasily, parodontal pouches are 3-4 mm deep, and contain purulentexudate. Orthopantomogram shows di-ffuse osteoporosis of alveolar process,resorption of interdental septa down to1/3 of their height. What is the most likelydiagnosis?

A. Exacerbation of chronic generalized Idegree periodontitisB. Chronic generalized I degree peri-odontitisC. Exacerbation of chronic generalized IIdegree periodontitisD. Chronic generalized II degree peri-odontitisE. Exacerbation of chronic catarrhal gingi-vitis

83. A 43-year-old patient consulted amaxillofacial surgeon about a cosmeticdefect in the right half of his nose. It isknown from the history that 7 monthsago he got a gunshot wound. Objecti-vely: there is a perforating defect 1,5 x2cm large in the right half of the nose. Theskin around the defect has not changed incolour. It was decided to use the "Indi-an"method of rhinoplasty. What tissue

should be taken for the defect correcti-on?

A. Forehead flapB. Cheek flapC. Infraorbital flapD. Shoulder flapE. Scapulohumeral flap

84. A 21-year-old patient complains ofconstant progressing throbbing pain in the27 tooth. Objectively: a large carious cavi-ty is filled with softened dentin, the toothcavity is closed. Probing of the cavity flooris painless, percussion causes acute pain,there is grade II tooth mobility. Palpationof the mucous membrane in the projecti-on of the root apex of the 27 tooth causespain. Radiological changes are absent.What is the most likely diagnosis?

A. Acute purulent periodontitisB. Exacerbation of chronic periodontitisC. Acute diffuse pulpitisD. Acute serous periodontitisE. Acute purulent pulpitis

85. A 25-year-old patient got a traumain the chin region. On the basis of X-rayhe was diagnosed with bilateral mentalfracture of mandible. Specify the directi-on of displacement of the minor fragment:

A. Downward and backwardB. Upward and forwardC. Upward and backwardD. Downward and forwardE. There is no displacement

86. 2 days ago a 12-year-old patientprsented with body temperature rise up to38, 0oC, on the second day he developed abilateral edema in the parotid region. Theskin over the edema was tense, of normalcolor. Palpation revealed soft, enlarged,painful parotid salivary glands. Salivationfrom the parotid ducts was abnormal, theduct orifices were hyperaemic. Palpationcaused pain in the angle of mandible, atthe top of mastoid bone, in front of theantilobium. What is the most likely di-agnosis?

A. MumpsB. Herzenberg pseudoparotitisC. Acute bacterial parotitisD. Purulent-necrotic parotitisE. Sjogren’s syndrome

87. A 38-year-old patient with chronicgeneralized periodontitis has beenreferred to orthopedic treatment. Objecti-vely: dentitions are without gaps, the 12,

Page 13: Krok 2 - 2012 Question Paper (Stomatology)

Krok 2 Stomatology 2012 13

11, 21, 22 teeth are pulpless and exhibit Igrade mobility. The other teeth are stable.What is the most aesthetic splint for theanterior teeth?

A. Mamlok splintB. Ring adhesion splintC. Soldered combined crownsD. Cap splintE. Brace

88. A 42-year-old patient has been hospi-talized with Le Fort II fracture of maxilla.Select an appliance for the treatment ofthis patient:

A. ZbarzhB. KulaginC. RudkoD. Penn-BrownE. Yadrova

89. An 8-year-old child presents with anedema of the submandibular region, themouth can be opened by 1,5 cm, furtheropening is difficult, body temperature is37, 6oC, mucogingival fold is vestibularlyflattened, hyperaemic and swollen. The 84and 85 teeth have fillings, their percussi-on is painless. The 84 tooth exhibits Idegree mobility. What is the most likelydiagnosis?

A. Acute odontogenic periostitisB. Acute odontogenic osteomyelitisC. Chronic odontogenic periostitisD. Chronic odontogenic osteomyelitisE. Exacerbation of chronic periodontitis

90. A 43-year-old patient complains aboutmobility of lower jaw teeth. Objectively:the dentition is intact. Tooth mobility isof I-II grade. It is planned to immobili-ze teeth by means of a removable splintcommon for the whole dentition. Whatstabilization will be provided by means ofthis splint?

A. CircleB. FrontalC. Frontal-lateralD. SagittalE. Transversal

91. A patient needs a clasp denture for thelower jaw. Objectively: dental formula is31, 32, 33, 34, 41, 42, 43, 44. The 44 totohis inclined toward the cheek. What type ofNey’s clasp should be used?

A. FourthB. FirstC. SecondD. ThirdE. Fifth

92. A 17-year-old man complains of acosmetic defect in form of light spots onthe teeth. In the area of his residence thefluoride concentration in drinking wateris at the rate of 1 mg/l. Objectively: vesti-bular surface of the 11,12, 21, 22, andtubercles of the 16, 26, 36 and 46 teethhave chalky spots with shiny surface thathave been present since the time of erupti-on. What is the most likely diagnosis?

A. Systemic hypoplasiaB. Multiple cariesC. Endemic fluorosisD. Enamel erosionE. Amelogenesis imperfecta

93. A 65-year-old patient complainsabout unsatisfactory fixation of completeremovable lamellar denture of his upperjaw. The denture was fabricated 6 yearsago. Objectively: balancing and poor fi-xation of complete removable denture ispresent. What is the reason of such condi-tion?

A. Atrophy of osseous base of theprosthetic bed tissuesB. Wear of artificial teethC. Bad hygiene of removable dentureD. Discoloration of basic plasticE. Loss of certain antagonists

94. A patient complains of heaviness inthe left section of his head, pain in the26 tooth. Objectively: the crown of the 26tooth is destroyed by 2/3 by caries, thetooth percussion is weakly positive. X-raypicture of paranasal sinuses shows defini-te unilateral dome-shaped veiling of theupper left maxillary sinus. On the X-raypicture of the 26 tooth the periodontal fi-ssure at the root apex is missing. What isthe most likely diagnosis?

A. Radicular cyst ingrown into the maxi-llary sinusB. RhinoantritisC. Odontogenic sinusitisD. Cyst of the maxillary sinus mucosaE. Malignant maxillary tumour

95. Preventive examination of a 5-year-old child revealed half-open mouth, di-fficult closing of lips, primary occlusion,4 mm sagittal gap, homonymous caninesand second molars. The upper dental arch

Page 14: Krok 2 - 2012 Question Paper (Stomatology)

Krok 2 Stomatology 2012 14

is V-shaped, the lower one is trapezoid.Both dental arches in primary occlusionshould have the following shape:

A. SemicircleB. SemiellipseC. ParabolaD. QuadrangleE. Triangle

96. A 46-year-old patient complains aboutpain and bleeding from the carious cavi-ty of her 27 tooth during eating. Previ-ously she had spontaneous pain. Exami-nation of the 27 tooth revealed a deepcarious cavity on the masticatory surfaceconsisting of red tissue, probing inducedpain and haemorrhage. What treatmentmethod should be chosen?

A. Vital extirpationB. Devital extirpationC. Devital amputationD. Vital amputationE. Biological method

97. A 25-year-old student complains of acarious cavity in the 22 tooth. The filli-ngs fell out 2 months ago. The tooth hadbeen treated before for pulpitis. Objecti-vely: there is a deep carious cavity withthe rests of filling on the medial surfaceof the 22 tooth. The crown of the 22 toothis dirty pink. X-ray shows a root canal fi-lled with the filling material by 1/2 of theroot length; in the region of the root apexthere is a well-defined focus of destructi-on of bone tissue 0,3x0,3 cm large. Whatis the most likely diagnosis?

A. Chronic granulomatous periodontitisB. Residual pulpitisC. Radicular cystD. Chronic fibrous periodontitisE. Chronic granulating periodontitis

98. A 35-year-old patient consulted adentist about extraction of the 14 toothbecause of exacerbation of chronic peri-odontitis following ineffective therapeutictreatment. What tools should be appliedfor extraction?

A. S-shaped forcepsB. Crown bayonet-shaped forcepsC. S-shaped forceps curved rightD. Straight forcepsE. Root bayonet-shaped forceps

99. A 32-year-old patient complains ofmouth soreness, body temperature rise upto 38, 5oC, indisposition. Such conditionhas occurred periodically for several years

after the patient had had a cold. Objecti-vely: lips are covered with haemorrhagiccrusts, hyperaemic mucous membrane oflips and cheeks has erosions covered withfibrinous films. Hypersalivation is present.What is the most likely diagnosis?

A. Multiform exudative erythemaB. Pemphigus vulgarisC. Herpes recidivicusD. Herpetiform Duhring’s dermatitisE. Stevens-Johnson syndrome

100. A 25-year-old man consulted a denti-st about extraction of the 18 tooth, it’scrown is destroyed by 1/2. The tooth wastreated more than once. During toothextraction the maxillary tuber was acci-dentally torn off. What actions should betaken?

A. To remove the fragment and stitch upthe woundB. To try to restore the fragment to itsplaceC. To restore the fragment to its place andfix it thereD. To remove the fragmentE. To remove the fragment and tamponthe wound

101. A 10-year-old child complains ofgingival pain and haemorrhage whichappeared two days ago after a cold.Objectively: the gingiva is edematic,hyperaemic, bleeds easily, painful onpalpation. The tips of gingival papillae aredome-shaped. What is the most likely di-agnosis?

A. Acute catarrhal gingivitisB. Chronic catarrhal gingivitisC. Hypertrophic gingivitisD. Ulcerative gingivitisE. Generalized periodontitis

102. A 53-year-old patient consulted adentist about pain and an ulcer thatturned up in the region of hard palateunder his partial removable denture. Hehasn’t used the denture for three weeks,but the ulcer does not heal or reducein size. Examination reveals a 2x1,5 cmlarge ulcer in the region of the alveolarprocess and hard palate at the level of themissing 14 tooth. The ulcer has evertedrolled edges and necrotic floor, it is denseand painful. Gingival and palatal tissuesaround the ulcer are infiltrated. What isthe most likely diagnosis?

Page 15: Krok 2 - 2012 Question Paper (Stomatology)

Krok 2 Stomatology 2012 15

A. Maxillary carcinomaB. Maxillary syphilisC. Maxillary tuberculosisD. Maxillary actinomycosisE. Decubital ulcer

103. A 13-year-old girl complains aboutfrequent falling out of a filling in the 21tooth. It is known from the anamnesis that2 years ago she underwent treatment onaccount of a dental trauma. Objectively:a transverse defect of 1/3 of the crown inthe 21 tooth. Percussion is painless. Toothcolour is unchanged. X-ray picture showsthat root canal is filled by 1 mm from theapex, filling material closely fits the wallsof root canal. What stomatological tacticsshould be chosen?

A. The defect should be restored withphotopolymerB. The root canal should be refilledC. The root apex should be resectedD. The tooth should be extractedE. The tooth should be crowned with anartificial crown

104. A 34-year-old male patient complai-ns of acute spasmodic pain in the regi-on of his upper jaw on the left that isgetting worse as affected by cold sti-muli. Toothache irradiates to the ear andtemple. He had acute toothache of the 37tooth one year ago, but he didn’t consulta dentist. Pain recurred three days ago.Objectively: the 37 tooth has a cariouscavity communicating with the dentalcavity. Probing of the opened cariouscavity is extremely painful. X-ray pictureshows widening of periodontal fissure atthe root apex of the 37 tooth. What is themost likely diagnosis?

A. Exacerbation of chronic pulpitisB. Exacerbation of chronic granulatingperiodontitisC. Exacerbation of chronic fibrous peri-odontitisD. Acute diffuse pulpitisE. Acute purulent pulpitis

105. During tooth extraction a 32-year-oldpatient presented with sudden weakness,pale skin, cold sweat, weak pulse, a signi-ficant AP drop (diastolic pressure - 40 mmHg). What complication developed in thepatient?

A. CollapseB. SyncopeC. Traumatic shockD. Anaphylactic shockE. Attack of stenocardia

106. A patient complains about pain in the45 tooth induced by cold, sour and sweetfood stimuli. The pain abates when thestimulus action is stopped. Objectively:there is a carious cavity on the masticatorysurface within mantle dentin filled withfood rests and softened dentin, overhangi-ng enamel edeges are chalky. What is thediagnosis?

A. Acute median cariesB. Chronic median cariesC. Acute superficial cariesD. Acute deep cariesE. Chronic deep caries

107. A 79-year-old female patientconsulted a prosthodontist about denturereplacement. The patient has a historyof a stroke. Objectively: acute irregularatrophy of the alveolar processes of bothjaws is present; mucous membrane ofthe oral cavity is dry and nonmobile.The previous dentures cannot be fixed.What is the most appropriate prostheticconstruction?

A. Dentures with elastic liningB. Dentures with extended bordersC. Dentures with metal basesD. Dentures with shortened bordersE. Implant-supported dentures

108. A 56-year-old patient consulted adental surgeon about the root extractionof the 17 toot. What kind of anaesthesiashould be applied?

A. Palatinal, tuberal anaesthesiaB. Palatinal, infraorbital anaesthesiaC. Infiltration, inscisor anaesthesiaD. Mandibular anaesthesiaE. Torus anaesthesia

109. A 23-year-old female patientcomplains of a pronounced limitation ofmouth opening, throat pain irradiating tothe ear. Objectively: body temperatureis 37, 9oC. The face is symmetrical, thecolour of skin has not changed. Palpati-on of the jaw causes acute pain, mouthopens by 1 cm. The patient had been gi-ven anasthesia by Berchet-Dubov. After itoral examination revealed an edema andhyperemia of the mucous membrane ofthe pterygomaxillary fold, there was aninfiltration painful on palpation. What is

Page 16: Krok 2 - 2012 Question Paper (Stomatology)

Krok 2 Stomatology 2012 16

the most likely diagnosis?

A. Abscess of pterygomaxillary spaceB. Abscess of alveololingual grooveC. Parapharyngeal space phlegmonD. Peritonsillar space abscessE. Submandibular phlegmon

110. Parents of an 8-year-old childcomplain about a painful formation inthe child’s oral cavity that obstructs foodintake. The same complaints were regi-stered two years ago. Mucous membraneof lateral tongue surface is hyperemic andedematic. There is an oval erosion over0,7 cm large covered with yellow greyi-sh deposit. Erosion edges are hyperemicand painful on palpation. The child has ahistory of chronic cholecystocholangitis.What is the most likely diagnosis?

A. Chronic recurrent aphthous stomatitisB. Erythema multiformeC. Behcet’s syndromeD. Stevens-Johnson syndromeE. Traumatic erosion

111. A 12-year-old child complains ofbody temperature rise up to 39, 8oC,weakness, headache and pain in throatgetting worse when swallowing. Objecti-vely: mucous membrane of gums isedematic, hyperemic. Tonsils are bri-ght red, hypertrophic, covered withyellow-gray deposit which does notextend beyond the lymphoid tissue andcan be easily removed. Submandibular,occipital lymph nodes are significantlyenlarged, slightly painful on palpation.Hepatosplenomegaly is present. Identi-fy the causative agent of this disease:

A. Epstein-Barr virusB. Bordet-Gengou bacillusC. Coxsackie virusD. Herpes virusE. Loeffler’s Bacillus

112. A 57-year-old patient complainsabout mobility of his metal-ceramic dentalbridge supported by the 33, 37 teeth.The bridge has been in use for 9 months.Objectively: X-ray picture shows alveolarprocess atrophy by 2/3 in the area of the33, and by 1/2 of root length in the area ofthe 37; there are pathological pockets, gi-ngivitis. What is the cause of pathologicalmobility of supporting teeth?

A. Functional shifting of supporting teethunder stressB. Injuring of circle ligament by crownedgesC. Devitalization of supporting teethD. Massive grinding off of hard tissuesE. Garland modelling in the precervicalregion

113. An orthodontist monitors a 4-year-old child with mouth breath. The child hasa history of adenotomy. Objectively: pri-mary dentition occlusion; the upper inci-sors overlap the lower ones by 1/3; distalsurfaces of the second temporary molarsare situated in the same vertical plane.What preventive device will help the childto give up the habit of mouth breath?

A. Standard Schonherr’s vestibular screenB. Vesibular and oral Kraus’ screenC. Frankel’s function regulatorD. Andresen-Haupl activatorE. Rudolph’s appliance

114. A 5-year-old boy complains of acarious cavity, periodic toothache, gumswelling. Objectively: masticatory surfaceof the 74 tooth has a deep cavity notcommunicating with the tooth cavity, coldstimuli, probing and percussion are pai-nless, mucous membrane is pale pink,there is a caicatrix from a fistula. X-ray pi-cture shows an ill-defined focus of bone ti-ssue destruction in the region of the rootsbifurcation. What is the most likely di-agnosis?

A. Chronic granulating periodontitisB. Exacerbation of chronic fibrous peri-odontitisC. Chronic fibrous periodontitisD. Exacerbation of chronic granulatingperiodontitisE. Chronic granulomatous periodontitis

115. A 13-year-old girl complainsof toothache increasing while biting.Objectively: the 36 tooth has a deep cari-ous cavity not communicating with thetooth cavity, the response to thermal sti-muli is painless, percussion is sharply pai-nful, mucous membrane is intact. X-raypicture shows no changes. What is themost likely diagnosis?

A. Acute serous periodontitisB. Acute purulent periodontitisC. Acute serous pulpitisD. Acute purulent pulpitisE. Exacerbation of chronic periodontitis

116. A 12-year-old patient complains

Page 17: Krok 2 - 2012 Question Paper (Stomatology)

Krok 2 Stomatology 2012 17

of gingival haemorrhage, tooth mobili-ty. He has has these presentations sincethe age of 4. Objectively: gingiva aroundall the teeth is hyperaemic, edematic,bleeds during instrumental examination.The teeth roots are exposed by 1/3 andcovered with whitish plaque. The teethare mobile. Dentogingival pockets are 4-5mm deep. External examination revealeddryness and thickening of the outer layerof skin on the palms, anterior third ofthe forearms, soles; there are skin cracks.What is the most likely diagnosis?

A. Papillon-Lefevre syndromeB. Hand-Schuller-Christian diseaseC. Niemann-Pick diseaseD. Letterer-Siewe diseaseE. Cyclic neutropenia

117. A 47-year-old patient came toan orthopaedic stomatology center wi-th complaints about missing tooth inthe frontal part of his upper jaw,cosmetic defect. Objectively: the occlusi-on is orthogenic, the 11 tooth is mi-ssing. Anamnesis data: the patient hadmyocardial infarction 3 months ago. Whatdenture should be applied for the timebeing?

A. Partial removable lamellar dentureB. Bridge denture suported by the 21 and12 teethC. Bridge denture suported by the 21 toothD. Clasp dentureE. Implant

118. Parents of a 6-year-old child appliedto a pedodontist for preventive examinati-on of their child. The oral cavity is saniti-zed. According to the parents, the childhas recently cut the 36 and the 46 tooth.What method of caries prevention shouldbe applied within 1,5-2 years after cuttingof the mentioned teeth?

A. Fissure hermetizationB. Fissure silveringC. Coating the teeth with fluorine lacquerFtorlakD. Remodentum solution applicationsE. Gargling with sodium fluoride

119. A 70-year-old patient is awaitingcomplete removable dentures for bothupper and lower jaws. Teeth placementwill be made by Vasilyev’s method. Whatteeth in the upper denture must not touchglass?

A. Lateral incisors and second molarsB. Central incisors and first molarsC. First and second premolarsD. Canines and first molarsE. Second premolars and first molars

120. A 20-year-old pregnant womancomplains of gingival enlargement,bleeding and pain during eating and toothbrushing. Objectively: gingival papillae onthe upper and lower jaw are hyperaemic,haemorrhagic, painfu, cover the crownsof teeth by 1/2. Scalloped contours ofmarginal gingiva are abnormal. What isthe most likely diagnosis?

A. Hypertrophic gingivitis, edematousformB. Acute catarrhal gingivitisC. Chronic catarrhal gingivitisD. Generalized periodontitisE. Hypertrophic gingivitis, fibrous form

121. A 55-year-old patient consulted adentist about a roundish tumour-likeformation of about 1 cm in diameterlocated within the vermilion border ofhis lower lip. Objectively: the tumour-like formation protrudes about 5 mmabove the vermilion border, it is dense andgrayish-red. The surface of the formationis covered with thin scales that can hardlybe removed. What is the most likely di-agnosis?

A. Verrucous precancer of the vermilionborder of lipB. Abrasive precancerous Manganotti’scheilitisC. Precancerous limited hyperkeratosis ofthe vermilion border of lipD. Bowen’s diseaseE. Erythroplasia of Queyrat

122. A 50-year-old patient has a defectof the lower dental arch. It is planned tomake an implant-supported bridge for itsrestoration. X-ray picture shows that theheight of the bone mass from projection ofmandibular canal up to the top of alveolarcrest is 2 cm. What type of implant shouldbe applied?

A. ThreadedB. Endodontic-endoosseousC. Plate-formD. SubperiostealE. Conical

123. External examination of a 7-year-oldchild revealed: thickening of nose bridge,semi-open mouth, dry lips. Mouth cornersare peeling. Anamnesis data: the child

Page 18: Krok 2 - 2012 Question Paper (Stomatology)

Krok 2 Stomatology 2012 18

sleeps with open mouth. Examination oforal cavity revealed no changes. What di-spensary group will this child fall into?

A. The secondB. The firstC. The thirdD. The fourthE. -

124. A 10-year-old child undergoes sani-tation of the oral cavity. The girl was foundto have chalky spots on the vestibularsurfaces in the precervical region of the21 and 12 teeth. Enamel surface is dull,smooth. Pain reaction to the temperaturestimuli is absent. What additional methodof examination is expected to confirm thediagnosis?

A. Vital stainingB. OrthopantomographyC. Intraoral roentgenographyD. ElectroodontodiagnosticsE. Ultraviolet stomatoscopy

125. Objective examination of a 10-year-old child revealed that the whole lowerlip was slightly hyperemic, infiltrated, dry,covered with small scales. Architectonicsof lips is changed. The child complains ofdryness and a feeling of tense lips, especi-ally in autumn and winter. The child had abad habit of lip sucking. What is the mostlikely diagnosis?

A. Meteorological cheilitisB. Allergic cheilitisC. Atopic cheilitisD. Exfoliative cheilitisE. Microbial cheilitis

126. A patient has got a traumatic fractureof mandible in the area of the missing 34,35 teeth with a slight displacement and adefect of alveolar part in the area of the34, 35 teeth. Other teeth on both lowerand upper jaws are intact. What splintwould be optimal in this case?

A. Tigerstedt’s splint with a spreadingcurveB. Plain splint crampC. Port’s splintD. Vasilyev’s splintE. Vankevich splint

127. A 30-year-old patient needs tohave his 26 tooth extracted because ofexacerbation of chronic periodontitis.Objectively: the crown of the 26 tooth isdecayed by 1/3. What forceps can be usedfor this tooth extraction?

A. S-shaped forceps with a projecting tipon the left beakB. S-shaped forceps with a projecting tipon the right beakC. Straight forcepsD. Straight elevatorE. S-shaped forceps without projecting tips

128. A 25-year-old male patient hasbilateral fracture of the lower jaw. Afragment in the region of the 44, 43, 42, 41,31, 32, 33, 34 teeth is displaced downwardand backward. What appliance should beused for the fragment reposition?

A. Post’s applianceB. One-jaw bite-guard splintC. Kurlyandsky’s appliance with leversD. Weber’s applianceE. Shur’s appliance

129. A 49-year-old patient consulted adental surgeon about the oral cavitysanation. He has an indication for theextraction of the 16 tooth. History: thetooth hasn’t been treated before, decayedwithin the last 4 years. Objectively: the16 tooth’s crown is destroyed by over2/3, the mucosa exhibits no pathologicalchanges. Which tool is required for thetooth extraction?

A. Bayonet root forcepsB. S-shaped forceps (right)C. Straight elevatorD. S-shaped closed-beak forcepsE. Crown bayonet forceps

130. A 9-year-old child complains of paincaused by sweet and sour food in an uppertooth on the left. Objectively: the 26 toothhas a carious cavity on the masticatorysurface within the enamel limits. What isthe optimal material to fill the 26 tooth?

A. СompositeB. Glass ionomerC. Silicophosphate cementD. Silicate cementE. Zinc phosphate cement

131. A patient complains aboutspontaneous pain in the area of his 15tooth he has been feeling for 2 days.Thermal stimuli make the pain worse, itsattacks last up to 30 minutes. Objecti-vely: there is a deep carious cavity inthe 15 tooth consisting of light softeneddentin, floor probing is painful in onepoint, reaction to the thermal stimuli ispositive, percussion is painless. Make a di-agnosis:

Page 19: Krok 2 - 2012 Question Paper (Stomatology)

Krok 2 Stomatology 2012 19

A. Acute local pulpitisB. Acute diffuse pulpitisC. Pulp hyperemiaD. Acute deep cariesE. Acute condition of chronic pulpitis

132. A 6-year-old child presents withweakness, pain in throat when swallowi-ng, body temperature rise up to 38, 0oC.Examination of the the oral cavityrevealed massive hyperaemia of themucous membrane of the soft palate,palatine arches, tonsils, uvula; there werealso single vesicles and erosions extremelypainful when touched. Regional lymphnodes are enlarged, painful on palpation.What is the most likely diagnosis?

A. HerpanginaB. ChickenpoxC. Mycotic anginaD. Infectious mononucleosisE. Diphtheria

133. A 35-year-old patient complainsof progressing throbbing pain in the 26tooth. Objectively: the 26 tooth has acarious cavity filled with softened denti-ne, tooth cavity is closed, probing of thecavity floor is painless, percussion causesacute pain. There is I grade tooth mobili-ty. Roentgenological changes are absent.What is the most likely diagnosis?

A. Acute purulent periodontitisB. Acute purulent pulpitisC. Acute serous periodontitisD. Exacerbation of chronic periodontitisE. Acute diffuse pulpitis

134. A patient complains of burning,itch and lower lip enlargement. He hasbeen suffering from this for a long ti-me. Objectively: the patient’s face isasymmetric due to the flattening ofnasolabial fold. His lower lip is edematic,of normal colour, painless on palpation.The patient has plicated tongue. What isyour provisional diagnosis?

A. Melkersson-Rosenthal syndromeB. Quincke’s edemaC. LymphangiomaD. HemangiomaE. Granulomatous Miescher’s cheilitis

135. A 50-year-old woman complainsabout a neoplasm on her lower lip onthe side of oral cavity that appeared amonth ago and has been slowly growingsince that. Objectively: there is a roundish,elastic, painless neoplasm inside the lower

lip. Mucous membrane hasn’t changed itscolour. Make a diagnosis:

A. Retention cyst of lower lipB. Lip abscessC. Lip papillomaD. Lip fibromaE. Lip lipoma

136. A 35-year-old patient complains of aneoplasm on the tip of the tongue whi-ch he hurts with his teeth. The neoplasmsometimes increases, and sometimesdecreases in size. Objectively: on the tipof tongue there is a roundish neoplasm0,5 cm in diameter with distinct bordersand broad base. The neoplasm is the samecolour as the mucosa of tongue. What isthe most likely diagnosis?

A. Papilloma of tongueB. Abscess of tongueC. Lipoma of tongueD. Hemangiofibroma of tongueE. Fibroma of tongue

137. A 35-year-old female patientconsulted a dentist about a painless,slowly growing neoplasm in the regi-on of the 11 and 12 teeth. Examinationrevealed that the tumour was light-pink,flat, adjacent to the teeth, had a pedicle.The tumour was up to 1,5 cm large, withsmooth surface and dense consistency. Itwas diagnosed as an epulis in the region ofthe 11 and 12 teeth. What form of epulisare these clinical findings typical for?

A. FibrousB. AngiomatousC. Giant-cellD. Pregnancy epulisE. -

138. As a result of a car accident a 45-year-old patient got an injury of his upperjaw. Examination revealed elongated andflattened face, profuse nasal haemorrhage,liquorrhea from the nose and ears. Theseclinical presentations are typical for thefollowing fracture of upper jaw:

A. Subbasal (Le Fort III)B. Subnasal (Le Fort I)C. Suborbital (Le Fort II)D. Bilateral fracture of zygomatic bonesE. -

139. A 7-year-old child has protrudingchin, the lower lip overlaps the upperone. There are diastemas and tremasbetween the lower incisors, the lower inci-sors overlap the upper incisors by 2/3 of

Page 20: Krok 2 - 2012 Question Paper (Stomatology)

Krok 2 Stomatology 2012 20

the crown height. First permanent molarsdemonstrate Angle’s class III relation.Sagittal gap is 3 mm. The correct doctor’stactics will be to:

A. Use Bruckl’s applianceB. Recommend a complex ofmyogymnastic exercisesC. Use Angle’s apparatusD. Use Bynin applianceE. Use Schwartz appliance

140. A 5-year-old child was found to havemissing upper molars. Lower incisors arein contact with the mucous membrane ofpalate. Specify the doctor’s tactics:

A. Fabricate a removable laminar dentureB. Examine the child every six monthsuntil the eruption of permanent teethC. Examine the child once a year until theeruption of permanent teethD. Fabricate an orthodontic appliance forthe treatment of closed biteE. Medical intervention is not needed

141. A child is 7 years old. He hasearly transitional dentition. There isovercrowding of the lower front teeth:the 42 and 32 teeth erupted orally witha complete lack of space. Make a plan oftreatment:

A. Serial consecutive extraction by HotzmethodB. Extraction of the 42 and 32 teethC. Extraction of the 41 and 31 teethD. Extraction of the 83 and 73 teethE. Extraction of the 84 and 74 teeth

142. A 14-year-old girl complains of indi-stinct pronunciation that showed up at theage of 14 after the acute respiratory vi-ral disease. Examination revealed normalface and normal teeth alignment, occlusaldisharmony was not found. Palpation di-dn’t reveal cleft palate. Uvula doesn’tmove during pronunciation of sounds, itspalpation does not cause gag reflex. Whatis the reason for indistinct pronunciationof sounds?

A. Paresis of the soft palate and uvulamusclesB. Adenoid vegetationsC. Palatal slitD. Hypertrophy of lingual tonsilE. Deformation of the bite

143. A 35-year-old patient complains ofa swelling in the sublingual region whi-ch impairs speech and causes discomfortduring eating. The swelling turned up

about a week ago. Objectively: there isa roundish bulge up to 3 cm in diameterin the right sublingual region. Mucousmembrane over it is not hyperaemic, thereis fluctuation symptom in the center. Whi-ch disease do these symptoms correspondwith?

A. Retention cyst of the right sublingualsalivary glandB. Retention cyst of the right submandi-bular salivary glandC. Acute sialoadenitis of submandibularsalivary glandD. Chronic sialoadenitis of sublingualsalivary glandE. Sjogren’s disease

144. During orthopaedic treatment a 47-year-old patient was given anaesthesia(with Ultracain DS forte). 20 minutes afterthe injection the patient presented wi-th hyperaemia of skin, headache, dizzi-ness, increased heart rate. The patient hadpreviously undergone dental treatmentwith the use of this anaesthetic. Whatcomplication developed in this patient?

A. AP riseB. Anaphylactic shockC. SyncopeD. CollapseE. Attack of stenocardia

145. A 35-year-old patient seeksprosthetic dentistry. Objectively: 18, 14,13, 12, 11, 21, 22, 23, 24, 28. The crownsconfining the defect are high and stable.To restore the integrity of the dentition itis planned to fabricate a one-piece claspdenture. What kind of mechanical fixatorshould be used?

A. Bar fixationB. Supporting-retaining claspsC. Retaining claspsD. Telescopic fixationE. Dentoalveolar clasps

146. A 42-year-old woman prsented to aprosthetic dentistry clinic seeking dentalprosthetics. Objectively: dental formula is

18 . . . . 13 12 11 21 22 23 . . . . 2848 47 46 45 44 43 42 41 31 32 33 34 35 36 37 .

The bite is deep, clinical crowns are low,survey line is not marked. The patientsuffers from epileptiform attacks. Whatprosthesis is indicated?

Page 21: Krok 2 - 2012 Question Paper (Stomatology)

Krok 2 Stomatology 2012 21

A. Partial removable lamellar prosthesiswith metal baseB. Dental bridgesC. Partial removable lamellar plasticprosthesis with retaining claspsD. Partial removable lamellar prosthesiswith supporting-retaining claspsE. Clasp denture

147. A 22-year-old patient presented to aprosthetic dentistry clinic because of mi-ssing of the 21 tooth, the 11 and 22 teethare intact. The 21 tooth was extracted 2months ago. What construction is mostsuitable in this case?

A. Two-stage implantationB. One-stage implantation with si-multaneous fabrication of metal-ceramiccrownC. Partial removable dentureD. Stamped-soldered denture supportedby the 11 and 22 teethE. Metal-plastic denture supported by the11 and 22 teeth

148. A 48-year-old patient has undergoneunilateral resection of the upper jaw. Heneeds a resection prosthesis. Objectively:the 21, 22, 23, 24, 25, 26 teeth are stable,hard palate is broad, flat. What connecti-on of clasps with the base will reduce theinstability of the resection prosthesis?

A. ArticularB. StableC. ElasticD. FixedE. Does not matter

149. At a dentist’s appointment a 12-year-old child inhaled a cotton turunda: hedeveloped noisy respiration with a whi-stling sound, sudden dyspnea, pallor alongwith pronounced cyanosis, sweating. Whatkind of urgent condition developed in thispatient?

A. Obstructive asphyxiaB. Dislocation asphyxiaC. Valvular asphyxiaD. Anaphylactic shockE. Collapse

150. A 39-year-old patient complains ofexperiencing pain in the region of the 21tooth for 2 days. It is known from the hi-story that the indicated tooth had beentreated before for caries. Objectively: the21 tooth is covered with metal-ceramiccrown, mucous membrane in apexprojection is edematic and hyperaemic.Percussion of the tooth is extremely pai-

nful. X-ray picture shows improperly filledroot canal. It is planned to remove the 21tooth crown. What kind of anaesthesiashould be given?

A. Field block anaesthesiaB. Infiltration anaesthesiaC. Intraligamentous anaesthesiaD. Application anaesthesiaE. Intraosseous anaesthesia

151. An 8-year-old child has a deep cari-ous cavity communicating with tooth cavi-ty on the distal-approximal masticatorysurface of the 75 tooth. Probing causespain. Percussion is painless. Cold watercauses slowly abating pain. The toothdecayed several months ago and wasn’ttreated. What treatment method wouldbe efficient in this case?

A. Devital amputationB. Biological methodC. Vital extirpationD. Vital amputationE. Devital extirpation

152. An 8-year-old child complains aboutpain in the 21 tooth that is getting worseduring biting down. A month ago a partof tooth crown broke off as a result ofa fall. The child didn’t consult a denti-st. Objectively: in the region of medialangle of the 21 tooth there is a crowndefect that makes up 1/3 of the crown’sheight. Tooth cavity is open, probing andthermal stimulus cause no pain. Percussi-on is acutely painful. Gum around the 21tooth is edematic and hyperaemic. Whatis the provisional diagnosis?

A. Acute condition of chronic periodonti-tisB. Acute condition of chronic pulpitisC. Acute serous periodontitisD. Acute purulent periodontitisE. Pulpitis complicated by periodontitis

153. A 12-year-old patient complainsabout an aesthetic defect. Objectively: thelower third of face is shortened, upperfrontal teeth overbite the lower teethby 3/3 of height, exhibit oral inclinati-on, lateral parts all along exhibit cusp-to-cusp relationship between the antagoni-sts; Angle’s class II malocclusion (joiningof the upper permanent molars) is alsopresent. Malocclusion is observed in thefollowing planes:

Page 22: Krok 2 - 2012 Question Paper (Stomatology)

Krok 2 Stomatology 2012 22

A. In sagittal and verticalB. In transversalC. In transversal and verticalD. In verticalE. In sagittal

154. A 52-year-old female patientcomplains of dry mouth, taste impai-rment, burning and pricking sensationsin the tongue that disappear during eati-ng but intensify at the end of the day. Forthe first time such problems arose 2 yearsago after a psychic trauma. She has a hi-story of anacid gastritis. Objectively: thegeneral condition is satisfactory, the pati-ent is restless, tearful. Oral mucosa is palepink, dry; filiform papillae on the dorsumof tongue are reduced. Pharyngeal reflexis dramatically reduced. There is segmetaldisturbance of facial skin sensation. Whatis the most likely diagnosis?

A. GlossodyniaB. Moller-Hunter glossitisC. Ganglionitis of sublingual ganglionsD. Chronic atrophic candidous glossitisE. Desquamative glossitis

155. A 40-year-old patient complainsabout difficult mastication as a result oflower jaw displacement. He has a historyof mental fracture 2 months ago. Objecti-vely: the 35, 36, 38 ... 45, 46 teeth are missi-ng. Remaining teeth are intact. The 43, 44,47, 48 teeth have no contact with antagoni-sts and their oral deviation makes up 1 cm.What is the optimal construction of lowerjaw denture?

A. Denture with double dentitionB. Metal-ceramic dental bridgeC. Soldered splint on ringsD. Removable lamellar dentureE. Adhesive denture

156. The department of maxillofacialsurgery admitted a patient who needsrepair of a post-traumatic defect of nosewing up to 3,0 cm in diameter. The traumaoccured six months ago. What kind ofgrafting is indicated in this clinical situati-on?

A. Grafting with chondrocutaneous flap ofthe auricleB. Grafting with local tissues of nasolabialor cheek regionsC. Grafting with pedicle flap of frontal andbuccal regionsD. Grafting with tubed pedicle (Filatov’s)flapE. Free grafting with dermal flap

157. During taking of impressions at adentist’s appointment a patient presentedwith the following symptoms: paroxysmalcough, cyanosis, vomiting, clapping soundduring inspiration. Make the diagnosis:

A. Aspiration of a foreign bodyB. Attack of bronchial asthmaC. Acute bronchitisD. TracheitisE. Hypersensitive gag reflex

158. At a dentist’s appointment a pati-ent complained of weakness, nausea,blackout, and then he lost consciousness.Make a diagnosis:

A. SyncopeB. ShockC. CollapseD. ComaE. Insult

159. During the preparation of a tooth apatient had an epileptic seizure. The sei-zure was arrested. What mistake did theorthopaedist make?

A. Didn’t collect complete history dataB. Did not apply one of the types of localanesthesiaC. Violated the rules of preparationD. Skipped psychological preparation ofthe patientE. Didn’t apply general anaesthesia

160. A 32-year-old patient needs tobe provided with metal-ceramic crownsfor the 12, 11, 21 and 22 teeth. Duri-ng the dental visit he is given infi-ltration anaesthesia with Ultracain DSanaesthetic. What elements enter into itscomposition?

A. 4% articaine with adrenalineB. 2% mepivacaine with adrenalineC. 4% articaine without a vasoconstrictoragentD. 3% mepivacaine without a vasoconstri-ctor agentE. 2% articaine with epinephrine

161. A 9-year-old boy has been diagnosedwith a complete dislocation of the 21tooth. The child got injured 20 hours ago.He has diabetes. Select a treatment tacti-cs:

Page 23: Krok 2 - 2012 Question Paper (Stomatology)

Krok 2 Stomatology 2012 23

A. The tooth cannot be preserved orreimplantedB. Reimplantation, fixation of the toothand further follow-upC. Root apex resection, reimplantation,fixation of the toothD. Filling of the root canal with amalgam,reimplantationE. Filling of the root canal with pastecontaining calcium, reimplantation, fixati-on of the tooth

162. Parents of a 7,5-year old child broughthim to the dentist for oral cavity sanitati-on. Objectively: DEF (for primary teeth)+ DMF (for permanent teeth) index =4, Green-Vermillion index = 2,5. Fissuresof the first permanent molars are open,intact, non-pigmented. What method ofprimary prevention of dental caries maybe appropriate in this case?

A. Non-invasive hermetizationB. Invasive hermetizationC. Application of fluoride varnishD. Application of antibacterial varnishE. Application of calcium gels

163. A 35-year-old patient complains of asignificant enlargement of the upper lipand eyelids, which developed within a fewminutes (during cleaning the house wi-th a detergent). Objectively: there is anedema of the upper part of face, upper lipand eyelids. Palpation is painless. Whatdisease are these symptoms typical for?

A. Angioneurotic Quincke’s edemaB. MacrocheilitisC. LymphedemaD. Melkersson-Rosenthal syndromeE. Glandular cheilitis

164. A 25-year-old HIV-infected patientpresented to a clinic of prosthetic denti-stry seeking prosthetic services. Whataseptic and antiseptic precautions shouldbe taken?

A. According to the schemeB. The impressions should be desinfectedby means of a quartz lampC. The patient should be refused appoi-ntmentD. No special precautions are requiredE. The orthopaedist should work in glovesand a mask

165. A 20-year-old patient got an injury.Objectively: the patient’s chin and lowerjaw up to the 34 and 45 teeth are missing.The 45, 46, 47, 48, 34, 35, 36, 37 teeth arestable. At what stage of medical evacuati-

on the patient will get special medical aid?

A. Specialized army surgical hospitalB. Battalion aid stationC. Regimental aid stationD. Separate medical detachmentE. Separate medical battalion

166. A 3-year-old child got an injury of theupper teeth as a result of a fall. Objecti-vely: crowns of the 51 and 61 teeth aredeep in the surrounding tissues with onlytheir cutting edge visible, the gingivalmargin is hyperaemia, edematic. What isthe treatment tactics?

A. Tooth extractionB. MonitoringC. RepositionD. Endodontic treatmentE. Anti-inflammatory therapy

167. A 25-year-old patient complains of alight brown spot in the upper front tooth.Objectively: the 23 tooth has a single li-ght brown spot in the precervical regi-on. Probing shows smooth surface. Thetooth is nonresponsive to cold and probi-ng. What is the most likely diagnosis?

A. Chronic initial cariesB. FluorosisC. Local enamel hypoplasiaD. Acute initial cariesE. Chronic superficial caries

168. A 45-year-old patient complains of arapidly growing formation on his lowerlip. Examination of the red border of li-ps revealed a greyish-red nodule with ahollow in the centre which is filled wi-th corneous masses that can be easilyremoved. The nodule is painless, mobi-le. What is your provisional diagnosis?

A. KeratoacanthomaB. PapillomaC. Nodulous verrucous precancer ofvermilion borderD. Basal cell carcinomaE. Circumscribed precanceroushyperkeratosis of vermilion border

169. A 35-year-old man complains aboutpain in the region of the 38 tooth, painfuldeglutition, difficult mouth opening. Whatanaesthesia method will be optimal duringoperation on account of pericoronaritis?

Page 24: Krok 2 - 2012 Question Paper (Stomatology)

Krok 2 Stomatology 2012 24

A. Conduction Bersche-Dubov’sanesthesiaB. Infiltration anaesthesiaC. General anaesthesiaD. Stem anaesthesiaE. Application anaesthesia

170. A 42-year-old patient complainsabout gingival pain, progressing gingi-val haemorrhage, increasing tooth mobi-lity, halitosis. Objectively: gums are evi-dently hyperaemic, extremely edematic,they bleed easily on palpation. Tooth rootsare exposed, parodontal pouches are 4-6mm deep, and contain purulent exudate,there is also supragingival and subgingivaldental calculus. II-III grade tooth mobi-lity is present. Orthopantomogram showsresorption of interdental septa down to1/2 of their height. What is the most likelydiagnosis?

A. Exacerbation of generalized II degreeperiodontitisB. Exacerbation of generalized I degreeperiodontitisC. Exacerbation of generalized III degreeperiodontitisD. Chronic generalized II degree peri-odontitisE. Chronic generalized III degree peri-odontitis

171. A female patient applied to theoral surgery department and underwentradical maxillary sinusotomy with plasticrepair of fistula through the alveolarsocket of the extrated 27 tooth. Infiltrationand all the peripheral block anaesthesiasof the left upper jaw were performed wi-th 6,0 ml of 2% lidocaine solution. 3 mi-nutes later the patient registered doublevision in her left eye, inability to closeit. Which of the performed anaesthesi-as is the reason for the above-mentionedpresentations?

A. InfraorbitalB. TuberalC. PalatinalD. IncisorE. Infiltration

172. A boy is 10 years old. His face issymmetric and proportional. He presentswith mouth breath. Examination of theoral cavity revealed saddle-like shape ofdental arches and high arched palate.Upper first molar relationship (Angle’skey to occlusion) remains intact. What isthe most likely diagnosis?

A. Narrowing of dental archesB. Distal occlusionC. Mesial occlusionD. Widening of dental archesE. Elongation of dental arches

173. A 51-year-old lecturer presented tothe orthopaedic department and complai-ned about painfullness and mobility of hisfrontal teeth of the lower jaw. Cervicesof the 42, 41, 31, 32 teeth are exposed,III grade mobility with deep pathologi-cal pockets is present. When would it bereasonable to fix dentures after dentalextraction?

A. On the day of dental extractionB. In 16-30 daysC. In 2-3 monthsD. In 4-6 monthsE. In 5-6 days

174. A 5-year-old child complains ofspontaneous pain in an upper jaw tooth onthe right that is getting worse at night andduring eating cold food. Objectively: the65 tooth has a deep cavity communicatingwith the tooth cavity. Probing is painful,percussion is painless. Cold water causeslong-standing pain. What is your provisi-onal diagnosis?

A. Exacerbation of chronic pulpitisB. Acute periodontitisC. Exacerbation of chronic periodontitisD. Acute serous pulpitisE. Acute purulent pulpitis

175. A 30-year-old patient complainsabout body temperature rise up to 39, 0oC,a roundish infiltrate on his upper lip,general weakness. He has been presentingwith these symptoms for 3 days. Objecti-vely: a roundish infiltrate in the regi-on of the upper lip 2,5 cm in diameter,the skin over the infiltrate is red with anecrotic core in the centre. The upper lipis hyperemic and edematic. What is themost likely diagnosis?

A. Furuncle of the upper lipB. Carbuncle of the upper lipC. Retention cyst of the upper lipD. Acute periostitis of the upper jawE. Acute glandular abscess

176. A patient complains aboutparoxysmal upper jaw toothache onthe left that is getting worse at night.Toothache intensifies also under stimulati-on and irradiates to the left eye andtemple. Similar attacks were noted threemonths ago, the patient didn’t undergo

Page 25: Krok 2 - 2012 Question Paper (Stomatology)

Krok 2 Stomatology 2012 25

any treatment. Objectively: the 25 toothhas a deep carious cavity communicati-ng with the tooth cavity. Probing causesacute pain at the point of communicati-on, vertical percussion is slightly pai-nful, horizontal one is painless. Mucousmembrane in the projection of root apexof the 25 tooth is unchanged, its palpati-on is painless. Thermal probe causesacute pain, the pain attack is long-lasting. Electroodontodiagnosis is 60 mi-croampere. X-ray picture shows a slightwidening of periodontal fissure at the rootapex of the 25 tooth. What is the most li-kely diagnosis?

A. Exacerbation of chronic pulpitisB. Acute generalized pulpitisC. Acute purulent pulpitisD. Acute purulent periodontitisE. Exacerbation of chronic periodontitis

177. A 48-year-old woman complains ofaching dull pain in the region of the leftTMJ, that is getting worse during eatingsolid food. The pain appeared about 2,5years ago. Objectively: mouth opening islimited, there is sideward deviation of jawduring mouth opening, TMJ is clicking.Examination of the oral cavity revealedsecondary partial adentia. X-ray pictureshows sclerosis of the cortical plate ofarticulat head and narrowing of cartilagespace. What is the most likely diagnosis?

A. Arthrosis of the TMJB. Chronic arthritis of the TMJC. Acute arthritis of the TMJD. Painful dysfunction of the TMJE. Exacerbation of chronic arthritis of theTMJ

178. An oral surgeon attended a 3-day-old child staying in the newbornpathology department. Objectively: bi-lateral hypogenesis of zygomatic bonesand orbits, antimongoloid slant ofpalpebral fissures, nonclosure of inferi-or eyelids, hypogenesis of mandible (birdface), deformity of auricles with a preauri-cular fistula on the right. What congenitaldisease does the child have?

A. Mandibulofacial dysostosisB. Oculocerebrorenal syndromeC. Oculoauriculovertebral dysplasiaD. Oculomandibulofacial syndromeE. Oculodentodigital syndrome

179. A pregnant 24-year-old womancomplains about emergence of severalnew carious cavities, falling out of old

fillings. Objective examination revealed:index of decayed, missing and filled teeth(DMF) = 16, Feodorov-Volodkina hygi-ene index is 3,3. Choose the optimalmaterial for carious cavity filling in thiscase:

A. Glass ionomer cementB. Silver amalgamC. Chemical-cure compositeD. Light-cure compositeE. Silicophosphate cement

180. A patient with a missile wound offace was delivered to the station of dentalaid group that was organized by orderof medical corps commander. What kindof aid can be rendered by the dental aidgroup?

A. Secondary careB. Professional careC. ConsultancyD. Dental health serviceE. Dental orthopaedic service

181. A 62-year-old patient has a medianfracture of mandible along with formati-on of a false joint. Objectively: dentalformula is 33, 34, 35, 36, 27, 47, 46, 45, 44,43. The teeth are intact, stable, with highcrowns. Fragment mobility is insignificant,there is no displacement. X-ray pictureshows a bone defect 0,8 cm large. Whatprosthesis is indicated?

A. Bridge-like prosthesis with a pivot pointB. Lamellar prosthesis without a pivotpointC. Lamellar prosthesis with Gavrilow’spivot pointD. Lamellar prosthesis with Oxman’s pivotpointE. Lamellar prosthesis with Weinstein’spivot point

182. After supercooling a 42-year-old pati-ent presented with headache in the leftfrontal region and left upper jaw. Objecti-vely: the face is symmetric, breathi-ng through the left nasal meatus isobstructed, seropurulent discharges arepresent. Palpation is slightly painful inthe infraorbital region as well as alongthe mucogingival fold in projection of the24, 25 teeth. Percussion of these teeth ispainless. The 24 tooth is filled. Mucuousmembrane of alveolar process has no visi-ble changes. X-ray picture shows reducedpneumatization of the left upper jaw si-nus. What is the provisional diagnosis?

Page 26: Krok 2 - 2012 Question Paper (Stomatology)

Krok 2 Stomatology 2012 26

A. Exacerbation of chronic odontogenicmaxillary sinusitisB. Acute periodontitis of the 24 toothC. Exacerbation of chronic periodontitis ofthe 24 toothD. Acute rhinogenous maxillitisE. Acute albuminous periostitis of the leftupper jaw

183. An 8,5-year-old child is apparentlyhealthy. The child complains of pain inan upper tooth on the left caused by atraumatic injury sustained three hoursago. Objectively: the crown part of the 21tooth is destroyed by 1/2, the pulp is redand significantly exposed, probing causesacute pain and bleeding. Percussion of the21 tooth is extremely painful. Choose themost efficient treatment method of the 21tooth:

A. Vital amputationB. Vital extirpationC. Devital amputationD. Devital extirpationE. Bioassay technique

184. A 23-year-old patient complains ofroot exposition, gingival haemorrhageduring tooth brushing, gum itch. Objecti-vely: there is supragingival and subgingi-val dental calculus. Gums are hyperaemic,edematic, pockets are 3,5 mm deep. X-raypicture shows resorption of interalveolarsepta by 1/3. What is the most likely di-agnosis?

A. Chronic generalized I degree peri-odontitisB. Chronic generalized II degree peri-odontitisC. Exacerbation of generalized I degreeperiodontitisD. Exacerbation of generalized II degreeperiodontitisE. II degree periodontitis

185. A 14-year-old child complains aboutacute spontaneous pain in an upper jawtooth on the right. The pain has beenlasting for 3 days, it is throbbing, irradi-ating to the temple, getting worse at ni-ght. Objectively: surface of the 15 toothexhibits a carious cavity within parapulpardentine. Dentine is softened, of greyishcolour. Probing of the whole cavity floor ispainful, percussion of the 15 tooth is pai-nless. What is the most likely diagnosis?

A. Acute purulent pulpitisB. Acute diffuse pulpitisC. Acute focal pulpitisD. Acute periodontitisE. Exacerbation of chronic periodontitis

186. A 9-year-old boy presents withface asymmetry due to the chin di-splacement to the left. When the thi-rd Il’ina-Marcosian diagnostic test isperformed, face asymmetry disappears.What is the most likely clinical form ofthis occlusal abnormality?

A. Habitual displacement of mandibleB. Ankylosis of the temporomandibularjointC. Unilateral hypoplasia of mandibleD. Bilateral narrowing of the maxillarydental archE. Unilateral narrowing of the maxillarydental arch

187. A 65-year-old male patient complai-ns of crepitation and clicking in bothtemporomandibular joints, pain inducedby displacement of the lower jaw to theright, ear noise, dry mouth, glossalgia.He has been using complete removableprosthesis of the lower jaw for 6 months.The patient denies having rheumatosis.Objectively: the lower third of faceis shortened, mental fold is stronglypronounced, mouth corners are lowered,there are angular fissures and cracks.Palpation reveals crepitatnt rale observedduring TMJ moving. What is the most li-kely diagnosis?

A. Costen’s syndromeB. Temporomandibular arthritisC. Temporomandibular arthrosisD. Temporomandibular dislocationE. Temporomandibular osteoarthritis

188. A 52-year-old patient complains ofteeth mobility, gingival haemorrhage. The44, 45, 34, 35 teeth exhibit I grade mobi-lity, the 43, 42, 41, 31, 32, 33 teeth exhi-bit II grade mobility. There is gingivaledema, peridontal pockets are 5-6 mmdeep. X-ray picture shows destruction ofbone tissue by 1/2 of root length. Thepatient has been diagnosed with generali-zed periodontitis of median severity. Whatorthopaedic construction is most efficientin this situation?

Page 27: Krok 2 - 2012 Question Paper (Stomatology)

Krok 2 Stomatology 2012 27

A. Elbrecht splintB. Plastic mouthguardC. Murray’s splintD. Mamlock splintE. Temporary Novotny splint

189. It is planned to lance an abscessof hard palate (it is localized close tothe 23, 24, 25 teeth) and to extract thecausative 24 tooth that had caused abscessdevelopment. What type of anaesthesia isindicated for this operation?

A. Infraorbital, palatinal and incisorB. Tuberal, infraorbital and incisorC. Tuberal, infraorbital and palatinalD. Tuberal and incisorE. Tuberal and palatinal

190. A 23-year-old patient complainsabout unpleasant sensations in the regi-on of the 12 tooth, X-ray picture shows awell-defined low-density area up to 8-10mm in diameter around the root apex ofthe 12 tooth. What method of surgery willbe the most efficient?

A. Root apex resectionB. Root amputationC. HemisectionD. Tooth removalE. Cystotomy

191. Examination of a 6-year-old boyrevealed enlarged lymph nodes in bothsubmandibular and cervical regions.Objectively: the 75, 84 and 85 teethare decayed, there are presentations ofcommissural cheilitis. According to theboy’s mother, he has been quickly getti-ng tired, sweating from the least physi-cal exercise, complaining about weaknessthroughout the last 2-3 months. He alsogave up training in a sports class. Whatplan of additional examination should beadopted?

A. Complete blood count, haematologistconsultationB. Biochemical blood test, endocrinologistconsultationC. Puncture biopsy, oncologist consultationD. Magnetic resonance tomography,immunologist consultationE. Pulmonary roentgenography,pulmonologist consultation

192. A 12-year-old girl undergoestreatment at the maxillofacial departmentfor nonodontogenic abscess of the leftsubmandibular region. Postoperativewound in the stage of granulation and epi-thelialization. What medications speed up

these processes?

A. Vishnevsky ointment, methyluracilointment, solcoseryl gelB. Laevosin, luronitum, trypsin ointmentsC. Levomecol, laevosin ointments etc.D. Trypisin, chymotrypsin, terrilytin etc.E. 0,5-1% dioxydin solution, 0,1-0,2%chlorhexidine solution

193. While conducting experiments inchemistry lesson a 14 year-old child gota traumatic shock of face. Objectively: theskin dehydrated in the affected region. Itis covered with thick dry crust with clearboundaries and peripheral hyperaemia,the crust is recessed into the skin. What isthe most likely traumatic agent?

A. AcidB. AlkalineC. RadiationD. ThermalE. Saline

194. A 7-year-old child has to undergoplastic surgery of the upper lip frenulum.What operation should be performed tolengthen the frenulum of the upper lip?

A. Dieffenbach graftingB. Thiersch grafting with local flapsC. Relaxing incisions of the mucosaD. Grafting with a pedicle flapE. Szymanowsky grafting with local flaps

195. The 47, 46, 45, 35, 36, 37, 38 teethof a 57-year-old patient are missing. Itis planned to make a clasp denture. The48 tooth inclines to the lingual side andforwards. On the lingual side of the 48tooth the border line is diagonal, on thebuccal side it runs on a level with gingivaledge. What type of Ney’s clasp should beapplied?

A. V type claspB. I type claspC. IV type clasp (reverse back-action)D. I-II type claspE. II type clasp

196. A patient complains about worsenedfixation and frequent breakages of parti-al removable lamellar denture for thelower jaw that has been in use for5 years. Objectively: alveolar part inedentulous regions is significantly atrophi-ed, the denture balances. What is the mostprobable cause of worsened fixation andfrequent breakages of the denture?

Page 28: Krok 2 - 2012 Question Paper (Stomatology)

Krok 2 Stomatology 2012 28

A. Alveolar process atrophyB. Improper care of dentureC. Denture using during sleepD. Consumption of solid foodE. Wear of artificial teeth

197. A 15-year-old patient has got atrauma. He complains of mandibular paingetting worse during swallowing, chewing,especially mouth opening. Objectively:face configuration isabnormal, there is alarge haematoma in the region of the ri-ght mandibular angle. Palpation of thisregion is painful. The mouth is half open,lower lip frenulum is shifted to the rightof the central line. Pressing upon the chincauses pain in the part of mandible on theright. What is the provisional diagnosis?

A. Fracture of mandibular angle on therightB. Bilateral fracture of mandibularbranchesC. Posterior unilateral dislocation ofmandibleD. Anterior unilateral dislocation ofmandibleE. Bilateral fracture of articular processes

198. A 12-year-old boy has been injured.In the region of the 44 and 45 teeth there ispathological displacement of the alveolarprocess and the body of the mandible,rupture of the mucous membrane of thealveolar process. What additional testsshould be done to specify the diagnosis?

A. Radiography of mandible in the frontaland lateral projectionsB. Radiography of skull in axillary projecti-onC. Radiography of skull in the frontalprojection and of mandible in ParmaprojectionD. Tomogram of mandibleE. Radiography of skull in the axillaryprojection

199. A 73-year-old patient consults adental orthopaedist about total edentia.Objectively: there is insignificant regularatrophy of the alveolar process ofmandible, the process is covered withmucous membrane of moderate elasticity.According to Keller’s classification, suchmanifestations are characteristic for thefolowing type of edentulous jaw:

A. FirstB. ThirdC. FifthD. SecondE. Fourth

200. Intraoral examination of a 5-year-oldchild revealed primary occlusion, tremasand diastemas, worn tubercles and cutti-ng surfaces of teeth. The distal surfacesof the second lower molars are anteriorto the distal surfaces of the second uppermolars. This stage of primary occlusion iscalled:

A. AgingB. Stable occlusionC. FormationD. EruptionE. There is no correct answer